You are on page 1of 49

www.UPSCPDF.com www.UPSCPDF.

com

1. Consider the following statements regarding 3. He was the pupil of Abu'l Fazal. Shahjahan
Madras Native Association: commissioned him to write a history of his
1. It represented mainly landed and reign modelled on the historic treatise of
business classes of Madras presidency. Akbarnama. He was the author of
2. It supported the state-sponsored Badshahnama. He went on to become the
missionary activities in the hope that official court historian of Shahjahan. The
they would receive an English above passage refers to which of the
education. following personalities?
Which of the statements given above is/are
(a) Abdul Hamid Lahori
correct?
(b) Muhammad Waris
(a) 1 only
(c) Sadullah Khan
(b) 2 only
(d) Abdus Samad
(c) Both 1 and 2
(d) Neither 1 nor 2
4. "It is a pooled investment fund that trades in
relatively liquid assets. It makes extensive
2. With reference to the various programmes/
use of complex trading and risk management
schemes implemented by the ministry of
techniques such as short selling, leverage,
minority affairs, consider the following
pairs: and derivatives to make quick returns."

Scheme Main Objective Which of the following is best described by


1. Naya : To provide education and skill the passage given above?
Savera training to the youth from (a) Hedge Funds
minority communities. (b) Venture Capital
2. Nai : To provide financial support to (c) Sinking Fund
Udaan the minority candidates (d) Angel Investment
clearing Prelims conducted by
Union Public Service 5. In the context of the recent ban of several
Commission, Staff Selection Chinese mobile apps by the Government of
Commission and State Public India, consider the following statements:
Service Commissions. 1. Powers to impose the ban were invoked
3. Nai : To empower the students under Information Technology Act,
Manzil belonging to minority 2000.
communities and prepare them 2. Individuals using banned Chinese apps
for competitive examinations.
are liable to be penalized under the
4. Nai : For Leadership Development
Indian Penal Code.
Roshni of minority women.
Which of the statements given above is/are
Which of the pairs given above are correctly
correct?
matched?
(a) 1 only
(a) 1 and 3 only
(b) 2 only
(b) 2 and 4 only
(c) Both 1 and 2
(c) 2, 3 and 4 only
(d) Neither 1 nor 2
(d) 1, 2 and 4 only
2 www.visionias.in ©Vision IAS
www.UPSCPDF.com
www.UPSCPDF.com www.UPSCPDF.com

6. Consider the following statements with 8. With reference to the Ecological Succession,
reference to the recently launched 'TULIP' consider the following statements:

portal: 1. An allogenic course of succession is


determined by the internal coactions of
1. It aims to provide internship
organisms and the environment.
opportunities to fresh graduates in Urban
2. Autogenic succession is largely
Local Bodies (ULBs) and Smart Cities mediated by competition for niche space
across the country. between species.
2. It is an initiative of the Ministry of Skill Which of the statements given above is/are
Development and Entrepreneurship. correct?
(a) 1 only
Which of the statements given above is /are
(b) 2 only
correct?
(c) Both 1 and 2
(a) 1 only
(d) Neither 1 nor 2
(b) 2 only
(c) Both 1 and 2 9. Consider the following statements:
(d) Neither 1 nor 2 1. The Constituent Assembly adopted
‘Elephant’ as its seal.
2. Sir B.N. Rau was appointed as a
7. Consider the following statements with
constitutional advisor to the assembly.
reference to the Infant Toddler and
3. Prem Behari Narain Raizada was the
Caregiver Friendly Neighbourhoods (ITCN)
calligrapher of the Indian Constitution.
Programme: Which of the statements given above is/are
1. The program aims to provide a certified correct?
course for young professionals in the (a) 1, 2 and 3
field of child care sensitization. (b) 1 and 3 only
(c) 2 and 3 only
2. It is an initiative of NITI ayog and the
(d) 2 only
Ministry of Women and Child
Development.
10. With reference to the Ganges Canal System,
3. It has been launched in light of the consider the following statements:
education crisis faced by the children of 1. It was funded by the British
migrant laborers during the COVID-19 government.

pandemic. 2. It was built in response to the Agra


famine of 1837-38.
Which of the following statements is/are
Which of the statements given above is/are
correct?
correct?
(a) 2 and 3 only (a) 1 only
(b) 1, 2 and 3 (b) 2 only
(c) 1 only (c) Both 1 and 2
(d) 3 only (d) Neither 1 nor 2

3 www.visionias.in ©Vision IAS


www.UPSCPDF.com
www.UPSCPDF.com www.UPSCPDF.com

11. National conference on education organized 14. Consider the following statements:
by congress in 1937 formulated a detailed 1. Osmium is the densest metal.
scheme of education called 'Wardha Scheme 2. Bromine is the only non-metal found in
of Basic Education with the help of Zakir the liquid state under normal conditions.
Hussain Committee. Which of the following 3. Technetium was the first man-made
were the features of the Wardha Scheme of element.
Basic Education? 4. All the rare earth metals are from the
1. Learning through activities. lanthanide series in the periodic table.
2. Free and compulsory education from 6 Which of the statement given above are
to 14 years. correct?
3. Providing basic handicraft training to (a) 2 and 4 only
children.
(b) 1 and 3 only
Select the correct answer using the code
(c) 1, 2 and 3 only
given below.
(d) 1, 2, 3 and 4
(a) 1 and 2 only
(b) 2 and 3 only
15. In the context of the Indian economy,
(c) 1 and 3 only
consider the following statements with
(d) 1, 2 and 3
respect to the term ‘Bilateral Netting’:
1. It refers to the single net claim of the
12. The National Authority for Chemical
two counterparties in a financial contract
Weapons Convention (NACWC) that acts as
to consolidate or offset all the claims
the national focal point for effective liaison
with the Organisation for the Prohibition of against each other.
Chemical Weapons (OPCW) was set up as 2. Bilateral netting is permitted under the
an office in the: Indian financial contracts.
(a) NITI Aayog 3. It is permitted only if both the
(b) Ministry of Chemicals and Fertilisers counterparties are regulated by the
(c) Cabinet Secretariat Reserve Bank of India.
(d) Prime Minister's Office (PMO) Which of the statements given above is/are
correct?
13. Consider the following statements regarding (a) 1 only
Aadhaar Authentication: (b) 1 and 2 only
1. It can be made mandatory by the Central (c) 3 only
as well as State Government for availing (d) 1, 2 and 3
government benefits.
2. Consent of the Aadhaar holder is 16. Which of the following is not correct with
mandatory for authentication. reference to the status of women in the
3. Authentication services can be availed Sangam age (3rd-century B.C.- 3rd century
by both the public and private entities A.D.)?
for verification of individual details. (a) Women were allowed to choose their
Which of the statements given above is/are life partners.
correct? (b) The practice of sati was prevalent in the
(a) 1 only higher strata of society.
(b) 2 and 3 only
(c) Widow remarriage was prevalent.
(c) 1 and 3 only
(d) Women excelled in education and arts.
(d) 1, 2 and 3
4 www.visionias.in ©Vision IAS
www.UPSCPDF.com
www.UPSCPDF.com www.UPSCPDF.com

17. Consider the following pairs: 20. In the context of the development of
Occupational Agents technical education in India during the
diseases British time, arrange them in chronological
1. Silicosis : crystalline silica dust order as to when they were established.
2. Asbestosis : Chrysotile dust 1. Calcutta College of Engineering
3. Black Lung : Coal dust
2. Engineering College at Roorkee
disease
3. Poona College of Engineering
Which of the pairs given above is/are
Select the correct answer using the code
correctly matched?
given below.
(a) 1 and 3 only
(a) 1-3-2
(b) 2 and 3 only
(b) 3-2-1
(c) 3 only
(d) 1, 2 and 3 (c) 1-2-3
(d) 2-1-3
18. Consider the following statements with
respect to the phenomena of 'solar 21. With reference to the 'Debt Service
minimum': Coverage Ratio (DSCR)' of a firm, consider
1. It is a part of the 11-year solar cycle the following statements:
marked by the decrease of sunspots and 1. The ratio is used to determine if a
solar flares. company will be able to sustain its debt
2. It marks the absence of the event of based on cash flow.
aurora borealis.
2. A DSCR of less than 1 means that the
Which of the statements given above is/are
company is able to pay its current debt
correct?
obligations without borrowing.
(a) 1 only
Which of the statements given above is/are
(b) 2 only
correct?
(c) Both 1 and 2
(a) 1 only
(d) Neither 1 nor 2
(b) 2 only

19. Recently seen in news e -SANTA is a/an (c) Both 1 and 2


(a) digital Trade dispute redressal (d) Neither 1 nor 2
mechanism of European Union and
India. 22. International Comparison Program (ICP)
(b) commodity trade platform of the Indian guided by the World Bank is aimed at the
Energy Exchange (IEX). international comparison of:
(c) electronic platform to connect aqua (a) Financial Inclusion
farmers and buyers in India. (b) Gross Domestic Product
(d) initiative of the Government of Uttar
(c) Poverty Alleviation
Pradesh to promote COVID-19
(d) Women Empowerment
vaccination.
5 www.visionias.in ©Vision IAS
www.UPSCPDF.com
www.UPSCPDF.com www.UPSCPDF.com

23. With reference to the Later Vedic Period, the 26. Consider the following pairs:
terms “satamana” and “krishnala” referred to Tribes Region
(a) Religious rituals 1. Limboos : Sikkim

(b) Coins (primitive) made of gold and 2. Birhors : Jharkhand

silver 3. Bharia : Uttarakhand


4. Raji : Madhya Pradesh
(c) God and goddesses
Which of the pairs given above is/are
(d) Republican and monarchical types of
correctly matched?
polity
(a) 1 and 2 only
(b) 3 and 4 only
24. What would happen if all the ice of the
(c) 1 only
Arctic region melts for some reason?
(d) 1, 2, 3 and 4
1. The earth's rotation will decrease.
2. Severe cold conditions will prevail in 27. Consider the following pairs:
Northern Western Europe. Organism Communication
Which of the above statements is/are not Signals
correct? 1. Dolphin : Magnetic signals
(a) 1 only 2. Olive Ridley : Acoustic Signals
(b) 2 only Turtles

(c) Both 1 and 2 3. Knife fish : Electric signals

(d) Neither 1 nor 2 Which of the pairs given above is/are


correctly matched?
(a) 1 only
25. Which of the following are included in the
(b) 1 and 3 only
original jurisdiction of the Supreme Court?
(c) 3 only
1. A dispute between the Government of
(d) 2 and 3 only
India and one or more States
2. A dispute regarding elections to either
28. Consider the following statements:
House of the Parliament or that of 1. An increase in the deposit rates will lead
Legislature of a State to an increase in the money multiplier.
3. Recovery of damages by a state against 2. If the money supply in an economy
the Centre. decreases, then the velocity of money
4. A dispute between two or more states will fall.
Select the correct answer using the code Which of the statements given above is/are
given below. correct?

(a) 1 and 2 only (a) 1 only

(b) 2 and 3 only (b) 2 only


(c) Both 1 and 2
(c) 1 and 4 only
(d) Neither 1 nor 2
(d) 3 and 4 only
6 www.visionias.in ©Vision IAS
www.UPSCPDF.com
www.UPSCPDF.com www.UPSCPDF.com

29. Consider the following statements in the 32. The Baikal-GVD' sometimes seen in the
context of the phenomena of Extirpation: news, is associated with:
1. It is a situation in which a species or a (a) Discovery of elementary particle of
population ceases to exist in a particular Higgs Boson from meteorite in Siberia
geographical location. region.
2. The phenomena are witnessed only (b) A nanohertz observatory in Baikal lake
because of the abiotic factors altering the region to detect seismic waves.
habitat of a species. (c) A project Of NASA & CERN to map
Which of the statements given above is/are subatomic particles called Fermions.
correct?
(d) An underwater neutrino telescope built
(a) 1 only
on Lake Baikal.
(b) 2 only
(c) Both 1 and 2
33. Consider the following statements regarding
(d) Neither 1 nor 2
the E-Courts project in India:
1. It is a pan-India project covering all high
30. With reference to National Defence Fund,
Courts and district courts.
consider the following statements:
2. It facilitates video-conferencing for
1. It is used for the welfare of the members
of the Para Military Forces and their judicial proceedings.
dependents. 3. It will be implemented by the High
2. Fund is administered by an Executive Courts in their respective jurisdiction.
Committee headed by Prime Minister. Which of the statements given above are
3. Defence Minister is the Treasurer of the correct?
Fund. (a) 1 and 2 only
4. Fund does not get any budgetary support. (b) 2 and 3 only
Which of the statements given above are (c) 1 and 3 only
correct? (d) 1, 2 and 3
(a) 1 and 3 only
(b) 2, 3 and 4 only 34. In the context of 'prior sanction' required to
(c) 1, 2 and 4 only prosecute army officers under the Armed
(d) 1, 2, 3 and 4 Forces Special Powers Act (AFSPA), 1958
consider the following statements:
31. Consider the following statements regarding 1. Prior sanction is required to prosecute
Central Waqf Council: army officers for all acts committed in
1. Central Waqf Council is a statutory the line of duty.
body. 2. Prior Sanction is required for the
2. Skill development of poor women is one
investigation.
of the objectives of the council.
3. Prior Sanction is not required for court-
3. It is under the administrative control of
martial.
the Ministry of Minority Affairs.
Which of the statements given above is/are
Which of the statements given above is/are
correct?
correct?
(a) 1 and 2 only
(a) 1 and 2 only
(b) 2 and 3 only
(b) 3 only
(c) 1 and 3 only (c) 3 only
(d) 1, 2 and 3 (d) 1, 2 and 3

7 www.visionias.in ©Vision IAS


www.UPSCPDF.com
www.UPSCPDF.com www.UPSCPDF.com

35. The Poverty and Growth Facility, an 38. Middle Income Group Scheme aims at:
institution that lends money to money to (a) providing free hostel facilities for
poor countries functions under the migrant workers.
administrative control of: (b) paid skill development for school
(a) World Bank dropouts.
(b) International Monetary Fund (c) providing subsidy for affordable housing.
(c) G20 (d) providing affordable legal services.
(d) Organisation for Economic Cooperation
and Development (OECD)
39. Which of the following statements is/are
correct with respect to the Agricultural and
36. Consider the following pairs:
Processed Foods Export Promotion
War Governor-
Development Authority (APEDA)?
General
1. It is a statutory body under the Ministry
1. Second Anglo : Lord Hastings
of Agriculture and Farmers' Welfare.
Maratha War
2. The Rice Export Promotion Forum
2. Third Anglo- : Lord Cornwallis
Mysore War (REPF) is set up under its aegis.
3. First Anglo-Sikh : Lord Dalhousie 3. It is mandated for registration and
War protection of Intellectual Property rights
Which of the pairs given above is/are with respect to Basmati Rice.
correctly matched? Select the correct answer using the code
(a) 1 and 2 only given below.
(b) 2 only (a) 2 only
(c) 1 and 3 only (b) 2 and 3 only
(d) 1, 2 and 3 (c) 1, 2 and 3
(d) 1 and 3 only
37. With reference to the Vesara style of temple
architecture, consider the following 40. Consider the following statements regarding
statements: National Policy for Rare Diseases, 2021:
1. The art of gopurams reached its climax 1. A rare disease is defined by WHO as
under the Vesara style.
one with a prevalence of one or less per
2. The Vesara style temples are
thousand population.
characterised by an open ambulatory
2. It provides for the setting up of a
passageway and decorated ceilings and
national hospital-based registry of rare
doorways.
diseases.
3. Chola and Pandyas dynasties
3. It envisages a crowdfunding mechanism
constructed most of their temples in the
from individuals and corporates.
Vesara style.
Which of the statements given above is/are Which of the statements given above is/are

correct? correct?
(a) 1 and 2 only (a) 1 and 2 only
(b) 2 only (b) 2 only
(c) 2 and 3 only (c) 1 and 3 only
(d) None (d) 1, 2 and 3

8 www.visionias.in ©Vision IAS


www.UPSCPDF.com
www.UPSCPDF.com www.UPSCPDF.com

41. In medieval times new devotional forms 44. With reference to the Ken Betwa Water
developed within the Hindu philosophical linkage project, consider the following
traditions. In that context, consider the
statements:
following statements:
1. Nath Panthis were followers of Shiva. 1. Ken-Betwa Link Project (KBLP) is first
2. Nath Panthis relied on yogic meditation project of the National Perspective Plan
in attaining immortality. for interlinking of rivers.
3. Shaktas were worshippers of universal
2. This project involves the transfer of
female energy.
Which of the statements given above are water from the Betwa to the Ken River
correct? through the construction of Daudhan
(a) 1 and 2 only Dam.
(b) 1 and 3 only
Which of the statements given above is/are
(c) 2 and 3 only
(d) 1, 2 and 3 correct?
(a) 1 only
42. Consider the following statements regarding (b) 2 only
various congress sessions during the
(c) Both 1 and 2
freedom struggle:
1. Vande Mataram was sung for the first (d) Neither 1 nor 2
time by Rabindranath Tagore at the 1896
congress session. 45. Consider the following faunal species:
2. The 1899 Lucknow session of the Indian
1. Nicobar bulbul
National Congress was presided over by
R.C. Dutt. 2. Nilgiri flycatcher
3. The last session of Congress before 3. Western tragopan
independence was held under the 4. Lesser flamingo
presidentship of Jawaharlal Nehru.
Which of the species given above are
Which of the statements given above is/are
correct? endemic to India?
(a) 1 only (a) 1 and 2 only
(b) 1 and 2 only (b) 1, 2 and 3 only
(c) 2 and 3 only
(c) 3 and 4 only
(d) 1, 2 and 3
(d) 1, 2, 3 and 4

43. Recently seen in the news,


Thumbimahotsavam is: 46. 'State of the Food Security and Nutrition in
(a) First ever dragonfly festival in Kerala.
the World' report is published by:
(b) Annual fair in the state of Tamil Nadu
(a) Solidarites International
commemorating Goddess Earth.
(c) An campaign of the Government of (b) Welthungerhilfe
India to mark the 100 years of Mapilla (c) Food and Agriculture Organisation
revolt.
(d) International Livestock Research
(d) A kind of a dance celebrated only during
Institute
times of Onam.
9 www.visionias.in ©Vision IAS
www.UPSCPDF.com
www.UPSCPDF.com www.UPSCPDF.com

47. Consider the following statements with 50. Moving from south to north, which of the
respect to 'Biosimilar Drugs': following is the correct order of the hills of
1. It has a structure that is highly similar to the North-East region of India?
but functionally different from a brand (a) Mizo hills, Mikir hills, Barail range,
name biologic drug. Dafla hills
2. It has more therapeutic effects and safety (b) Barail range, Mizo hills, Dafla hills,
features than its generic drug Mikir hills
counterpart. (c) Mikir hills, Mizo hills, Barail range,
Which of the statements given above is/are Dafla hills
correct? (d) Mizo hills, Barail range, Mikir hills,
(a) 1 only
Dafla hills
(b) 2 only
(c) Both 1 and 2
51. Consider the following statements with
(d) Neither 1 nor 2
respect to the Director General of Foreign
Trade (DGFT):
48. Consider the following pairs:
1. It is an attached office under the
Non-Banking Regulated by
Ministry of Commerce and Industry.
Financial
2. It is responsible for formulating and
Companies
1. Core Investment : Ministry of implementing the foreign trade policy of
Company (CIC) Corporate Affairs India.
2. NIDHI Company : Reserve Bank of 3. It deals with the cases of anti-dumping
India and trade related safeguard measures.
3. Chit fund : Securities and Which of the statements given above are
company Exchange Board correct?
of India (a) 1 and 2 only
Which of the pairs given above is/are not (b) 2 and 3 only
correctly matched? (c) 1 and 3 only
(a) 1 only (d) 1, 2 and 3
(b) 1 and 2 only
(c) 2 and 3 only 52. “This island off the coast of South America
(d) 1, 2 and 3 is extremely fortunate in having the world’s
most concentrated single source of natural
49. What is/are the possible effects of using manure. This rainless and desert island is
hundred octane rating petrol? home to millions of birds that live entirely
1. Internal combustion engines will have on sea fish. The droppings of the sea birds
the highest resistance to knocking. have been in use for several hundred years as
2. It could increase CO2 emissions from
valuable manure for a variety of crops.”
petrol engine vehicles.
Which of the following islands is being
Select the correct answer using the code
referred to in the above passage?
given below.
(a) Galapagos island
(a) 1 only
(b) Guano island
(b) 2 only
(c) Easter island
(c) Both 1 and 2
(d) Falkland island
(d) Neither 1 nor 2
10 www.visionias.in ©Vision IAS
www.UPSCPDF.com
www.UPSCPDF.com www.UPSCPDF.com

53. Consider the following statements with 56. With reference to the Global Centre for
respect to Himalayan Serow: Nuclear Energy Partnership, consider the
1. Himalayan Serow is a vulnerable following statements:
category IUCN mammal.
1. It is an initiative of the International
2. Himalayan Serow gets the protection of
Atomic Energy Agency (IAEA).
Schedule 1 under the Wildlife Protection
2. The center of the global partnership has
Act, 1972.
3. Recently it was spotted in Cold desert been set up in Haryana.
National Park and Manas National park. 3. It aims to train manpower in the field of
Which of the statements given above is/are nuclear safety and radiological safety.
correct? Which of the statements given above is/are
(a) 1 and 2 only correct?
(b) 1 and 3 only (a) 1, 2 and 3
(c) 3 only
(b) 2 and 3 only
(d) 1, 2 and 3
(c) 1 only
(d) 1 and 3 only
54. Consider the following pairs:
Folk Dance State
1. Charba : Himachal Pradesh 57. Consider the following statements about
2. Paika : Odisha National Pension Scheme (NPS):
3. Jhoomar : Gujarat 1. Its objective is to provide a fixed
Which of the pairs given above is/are minimum pension to all beneficiaries at
correctly matched? the age of 60.
(a) 1 and 3 only
2. Both Indian residents and NRIs are
(b) 2 only
eligible for applying under the scheme.
(c) 1 and 2 only
(d) 1, 2 and 3 Which of the statements given above is/are
correct?
55. With reference to High-Security Registration (a) 1 only
Plate (HSRP), consider the following (b) 2 only
statements: (c) Both 1 and 2
1. The high-security number plates are (d) Neither 1 nor 2
made of aluminium and come with
reflective tapes.
58. ‘Tchay-e-gard Shikar’ sometimes seen in the
2. The plates are tamper-proof equipped
news, refers to:
with a chromium-based and self-
destructive hologram. (a) the traditional form of fishing popular in
3. The permanent identification number Kashmir Valley.
(PIN) of a minimum of 10 digits is laser (b) the traditional art of woodcarving
branded on the plate. popular in Ladakh Region.
Which of the statements given above is/are (c) a special pine craft that got GI tag status
correct?
popular in highland region of Sikkim.
(a) 1 only
(d) an Eco- friendly method to substitute
(b) 2 and 3 only
Slash and Burn Farm practices in North
(c) 1 and 2 only
(d) 1, 2 and 3 East India.
11 www.visionias.in ©Vision IAS
www.UPSCPDF.com
www.UPSCPDF.com www.UPSCPDF.com

59. With reference to the Survey of India, 62. Which of the following is correct with
consider the following statements: respect to the Finance Commission of India?
1. It is the National Mapping Agency under (a) The President specifies the qualifications
the Ministry of Earth Sciences. of the chairman and members of the
2. It is responsible for fixing the external commission.
boundary of India. (b) The recommendations made by the
3. It makes regular observations and Finance Commission are only of
understanding of seabed characteristics advisory nature.
of the Indian Ocean.
(c) The Finance Commission is required to
Which of the statements given above is/are
make recommendations to Parliament on
correct?
principles that should govern the grants-
(a) 1 and 2 only
in-aid to the states by the Centre.
(b) 2 only
(d) It is constituted by the Parliament every
(c) 3 only
fifth year or at such earlier time as
(d) 1, 2 and 3
considered necessary.

60. What is 'miracle of aggregation' in politics?


63. ‘RuTAG’ sometimes seen in the news is:
(a) A situation where number of small
(a) a mechanism to achieve rural
parties join together to form the
government. advancement through S&T interventions.

(b) A situation where large numbers of (b) a mechanism of geo-tagging of assets

uninformed people balance each other created under MGNREGA.


out when voting. (c) a bar code through which toll is
(c) Projecting a candidate with the sole automatically deducted at toll plaza.
purpose of diversion of votes of a (d) a communication satellite launched by
specific social group. ISRO.
(d) A situation where the voting in the state
election is influenced by national issues. 64. Which of the following sectors is/are not
included in the Index of Industrial
61. Which of the following statements correctly Production?
explains Biomimetics? 1. Electricity
(a) The branch of biotechnology that studies 2. Construction
the mode of life of organisms in their 3. Mining
natural habitat. 4. Water and Gas Supply
(b) Imitating nature using nanofabrication
Select the correct answer using the code
techniques for commercial applications.
given below.
(c) Culturing microorganisms to express
(a) 1 and 2 only
fluorescent proteins.
(b) 2 only
(d) The process to design special class of
(c) 1 and 3 only
antibodies commonly known as
(d) 2 and 4 only
"Monoclonal antibodies".
12 www.visionias.in ©Vision IAS
www.UPSCPDF.com
www.UPSCPDF.com www.UPSCPDF.com

65. Saura paintings, mural paintings made by a 68. Recently seen in the news "Maitri Setu" is
tribe called Saura tribe, belong to built over the river
(a) Odisha (a) Feni
(b) Andhra Pradesh (b) Brahmaputra
(c) Rajasthan (c) Ganga
(d) Himachal Pradesh
(d) Hooghly

66. Consider the following statements with


69. Consider the following statements regarding
respect to Sixth National Report on
the Brahmi script of India:
Biodiversity (NR6):
1. It is the oldest writing script system
1. It is an assessment report of 12 'National
Biodiversity Targets' developed within developed in the Indian subcontinent.

Aichi biodiversity targets. 2. It is usually written from left to right.


2. India has successfully achieved its 12 3. All the Ashokan inscriptions were
National Biodiversity Targets (NBT) composed in the Brahmi Script.
well before the deadline. Which of the statements given above is/are
Which of the statements given above is/are not correct?
correct? (a) 1 only
(a) 1 only (b) 3 only
(b) 2 only
(c) 1 and 3 only
(c) Both 1 and 2
(d) 2 and 3 only
(d) Neither 1 nor 2

70. With reference to 'mutual delegation of


67. Consider the following statements regarding
powers between Centre and States, consider
labour interests in the twentieth century:
1. The Government of India Act, 1919 the following statements:

provided for labour representation in 1. President can entrust to a state


legislative councils. government any of the executive
2. The 1919 congress session urged functions of the Centre even without the
provincial congress committees to consent of the state government.
promote labour unions. 2. Governor of a State may, with the
3. Bombay Trade Disputes Act, 1938 consent of Central Government, entrust
enacted by the Bombay provincial to the central government any of the
congress government was considered to
executive functions of the state.
be against labour interests.
Which of the statements given above is/are
Which of the statements given above are
correct?
correct?
(a) 1 only
(a) 1 and 2 only
(b) 2 only
(b) 2 and 3 only
(c) 1 and 3 only (c) Both 1 and 2

(d) 1, 2 and 3 (d) Neither 1 nor 2


13 www.visionias.in ©Vision IAS
www.UPSCPDF.com
www.UPSCPDF.com www.UPSCPDF.com

71. Consider the following statements regarding 74. Consider the following statements:
President's Bodyguard: 1. Arachnids are insects that belong to
1. It was first raised by Warren Hastings. Phylum Coelenterata.
2. The most conspicuous features of
2. It is senior-most in the order of
Arachnids are three pairs of legs and
precedence of the units of the Indian
antennae.
Army.
Which of the statements given above is/are
3. It comprises of members only from 3 correct?
castes — Jats, Rajputs & Jat Sikhs. (a) 1 only
Which of the statements given above are (b) 2 only
correct? (c) Both 1 and 2
(a) 1 and 2 only (d) Neither 1 nor 2
(b) 2 and 3 only
75. What is/are unique about the Kharai camel, a
(c) 1 and 3 only
breed found in India?
(d) 1, 2 and 3
1. It is found in the Gujarat region of India.
2. They have the special ability to survive
72. With reference to the Vigyan Jyoti Program, on both, dry land and in the sea.
consider the following statements: 3. It feeds on saline plants.
1. The program is a collaboration between Select the correct answer using the code
ISRO and AICTE. given below.
(a) 1 only
2. The program aims to promote STEM
(b) 1 and 2 only
(Science Technology Engineering
(c) 1, 2 and 3
Mathematics) learning in girl students. (d) None
Which of the statements given above is/are
correct? 76. Consider the following pairs:
(a) 1 only Biogeographic Definition
(b) 2 only Rule
(c) Both 1 and 2 1. Rapoport's : Species at higher
Rule latitudes tend to have
(d) Neither 1 nor 2
smaller latitudinal
ranges.
73. Consider the following statements with 2. Bergmann's : Organism body size is
respect to Power Sector in India: Rule large in cold climates
1. Except Atomic Energy, 100% FDI is and small in warm
allowed in the generation of electricity climates.
from all the sources. 3. Gloger's : Birds in more humid
2. India is the sixth-largest producer of rule habitats tended to be
darker than in regions
electricity in the world.
with higher aridity.
Which of the statements given above is/are
Which of the pairs given above are correctly
not correct? matched?
(a) 1 only (a) 1 and 2 only
(b) 2 only (b) 2 and 3 only
(c) Both 1 and 2 (c) 1 and 3 only
(d) Neither 1 nor 2 (d) 1, 2 and 3

14 www.visionias.in ©Vision IAS


www.UPSCPDF.com
www.UPSCPDF.com www.UPSCPDF.com

77. With reference to the Gandhi Peace Prize, 80. Consider the following statements about
consider the following statements: Asafoetida (Heeng):
1. The award can be conferred upon any 1. It is a perennial spice crop.
person irrespective of her nationality. 2. It is endemic to Iran and Afghanistan.
2. It cannot be awarded to an individual Which of the statements given above is/are
posthumously. correct?
3. The award is open to both individuals (a) 1 only
and organizations for their exemplary (b) 2 only
work promoting non-violence and (c) Both 1 and 2
gandhian methods. (d) Neither 1 nor 2
Which of the following statements is/are
correct? 81. Consider the following statements with
(a) 3 only respect to Invest India:
(b) 1 and 2 only 1. It was established as a non-profit venture
(c) 1, 2 and 3 to promote foreign investment in India.
(d) 1 and 3 only 2. It functions under the administrative
control of the Ministry of Commerce
78. Consider the following statements in the and Industry.
context of the recently launched Poshan 3. The Central Government owns 51% of
Gyan: the shares in Invest India.
1. It is launched by the Ministry of Health Which of the statements given above are
and Family Welfare. correct?
2. It is a national digital repository on (a) 1 and 2 only
health and nutrition. (b) 2 and 3 only
Which of the statements given above is/are (c) 1, 2 and 3
correct? (d) 1 and 3 only
(a) 1 only
(b) 2 only 82. Siddhantashiromani, a major treatise on
(c) Both 1 and 2 mathematics, was authored by:
(d) Neither 1 nor 2 (a) Brahmgupta
(b) Mahaviracharya
79. Consider the following statements with (c) Narayan Pandit
reference to the Atal Innovation Mission (d) Bhaskaracharya
(AIM)-PRIME:
1. It aims to promote science-based deep 83. Consider the following statements with
technology start-ups in India. respect to CHUNAUTI programme recently
2. The program shall be open only to the launched by the Union Government:
students enrolled under different Atal 1. It was launched by the Ministry of
Incubation Centres across the nation. Commerce and Industry.
3. It is a joint initiative of Atal Innovation 2. It aims to further boost start-ups and
Mission (AIM) and Bill & Melinda software products, especially in India’s
Gates Foundation (BMGF). tier-II towns.
Which of the following statements is/are Which of the statements given above is/are
correct? correct?
(a) 1 and 2 only (a) 1 only
(b) 3 only (b) 2 only
(c) 2 and 3 only (c) Both 1 and 2
(d) 1 and 3 only (d) Neither 1 nor 2
15 www.visionias.in ©Vision IAS
www.UPSCPDF.com
www.UPSCPDF.com www.UPSCPDF.com

84. With reference to the 'Aahaar Kranti' 87. Consider the following:
Mission, consider the following statements: 1. Litmus
1. The mission has been launched to create 2. Red Cabbage
awareness about the use of iodized salt. 3. Onion
4. Turmeric
2. It has been launched by the Ministry of
Which of the above can be used to indicate
Food Processing in collaboration with
the presence of acid or base in a solution?
Vijnana Bharati (Vibha). (a) 1 and 2 only
3. The movement will work on applying (b) 1, 2 and 3 only
Ayurveda- based nutrition knowledge to (c) 3 and 4 only
practice. (d) 1, 2, 3 and 4
Which of the statements given above is/are
correct? 88. With reference to delimitation, consider the
(a) 3 only following statements:
1. The present delimitation of
(b) 1 and 2 only
parliamentary constituencies has been
(c) 1, 2 and 3
done on the basis of 2001 census figures
(d) 2 only by 87th Amendment Act.
2. Parliament is empowered to determine
85. Consider the following statements with the authority and the manner in which
respect to the Shukrayaan Mission of India: delimitation is to be made.
1. Shukrayaan will be the ISRO's first Which of the statements given above is/are
mission to Venus. correct?
(a) 1 only
2. It will map Phosphine gas, an important
(b) 2 only
biomarker in Venus atmosphere.
(c) Both 1 and 2
Which of the statements given above is/are
(d) Neither 1 nor 2
correct?
(a) 1 only 89. Which of the following statements is/are
(b) 2 only correct regarding the National Bank for
(c) Both 1 and 2 Financing Infrastructure and Development
(d) Neither 1 nor 2 (NBFID)?
1. It is a statutory institution that aims for
the development of long-term
86. Consider the following pairs:
infrastructure financing in India.
Mineral World region
2. It can raise money in Indian rupees as
1. Iron : Mesabi range
well as in foreign currencies.
2. Kinta Valley : Copper 3. It can lend or invest, directly or
3. Highveld : Gold indirectly in infrastructure projects
4. Chuquicamata : Tin located entirely or partly in India.
Which of the pairs given above is/are Select the correct answer using the code
correctly matched? given below.
(a) 1 and 3 only (a) 1 and 2 only
(b) 1 and 3 only
(b) 3 only
(c) 3 only
(c) 2 and 4 only
(d) 1, 2 and 3
(d) 1, 2, 3 and 4
16 www.visionias.in ©Vision IAS
www.UPSCPDF.com
www.UPSCPDF.com www.UPSCPDF.com

90. In the context of the monetary policy of 93. Consider the following statements about the
India, which of the following best describes Mega Food Park Scheme:
the term 'Corridor'? 1. It aims to link agricultural production to
the market by bringing together various
(a) Area between the lower reverse repo rate
stakeholders.
and the upper ceiling rate of Marginal
2. It is implemented by the National
Standing Facility (MSF) rate.
Institute of Food Technology,
(b) Area between the lower reverse repo rate Entrepreneurship and Management.
and the upper ceiling rate of repo rate. 3. This scheme is a sub-component of the
(c) Area between the Cash Reserve Ratio Pradhan Mantri Kisan Sampada Yojana
(CRR) and Statutory Liquidity Ratio (PMKSY).
(SLR). Which of the statements given above are
correct?
(d) The monetary transition area due to
(a) 1 and 2 only
change in monetary policy over a period
(b) 2 and 3 only
of time.
(c) 1 and 3 only
(d) 1, 2 and 3
91. Mode-4 under the General Agreement on
Trade in Services (GATS) refers to: 94. Ratoon cropping is growing a fresh crop
(a) supply of a service of one country to the from the stubbles or suckers of the plant crop
service consumer of any other country. without replanting. Which of the following
(b) services provided by a service supplier crops are cultivated by using ratoon cropping
method?
of one country in the territory of any
1. Sorghum
other country.
2. Pineapple
(c) services provided by a service supplier 3. Banana
of one country in the territory of any 4. Sugarcane
other country through commercial Select the correct answer using the code
presence. given below.
(d) presence of persons in the territory of (a) 1 and 3 only
another for the purpose of providing a (b) 2 and 4 only
(c) 2, 3 and 4 only
service.
(d) 1, 2, 3 and 4

92. Which of the following statements is/are 95. Under Hazardous and Other Wastes
correct regarding Peaty/marshy soils? (Management and Transboundary
1. They originate as a result of the Movement) Rules 2016, which of the
accumulation of large amounts of following items are prohibited for import?
organic matter in the soil. 1. E-waste
2. They are highly acidic. 2. Solid plastic waste
3. Waste edible fats
3. These soils in Kerala are known as Kari.
4. Metal scrap
Select the correct answer using the code
Select the correct answer using the code
given below. given below.
(a) 1 and 2 only (a) 1 and 4 only
(b) 3 only (b) 2 and 3 only
(c) 1, 2 and 3 (c) 1 and 2 only
(d) 2 and 3 only (d) 1, 2, 3 and 4
17 www.visionias.in ©Vision IAS
www.UPSCPDF.com
www.UPSCPDF.com www.UPSCPDF.com

96. Consider the following statements regarding 99. GPS Aided GEO Augmented Navigation
Gandhiji's participation during the Quit India
(GAGAN) is an Indian Satellite-Based
Movement:
1. Gandhiji's fast during this movement Augmentation System (SBAS). It is
was directed against the colonial designed to assist which of the following
government's repressive acts and arrest
modes of transport?
of thousands of congressmen.
2. Gandhiji was released from jail in 1944 1. Aviation
on the condition that he would condemn 2. Maritime
violence.
3. Land transport
Which of the statements given above is/are
correct? Select the correct answer using the code
(a) 1 only given below.
(b) 2 only
(a) 1 only
(c) Both 1 and 2
(d) Neither 1 nor 2 (b) 2 and 3 only
(c) 1 and 3 only
97. Recently seen in the news MANAS App is
related to: (d) 1, 2 and 3
(a) An application developed to augment
mental well being.
100. Consider the following statements with
(b) An initiative of IIT Delhi to monitor the
availability of Remdesivir. respect to the cultural history of India:
(c) Application developed to monitor the 1. Dhamma, Sangha, Tripitakas are
recently seen Serow in Manas wildlife
considered as the three jewels (Triratnas)
sanctuary.
(d) A digital platform for the fishermen of of Buddhism.
the state of Tamil Nadu and Kerala. 2. All the three Tripitakas were written in
the Pali language.
98. Consider the following statements with
respect to the Amrabad Tiger reserve in 3. Buddhism was divided into Mahayana
India: and Hinayana Buddhism during the
1. It is the second-largest Tiger protected
reign of Ashoka.
area in India.
2. It lies in the Nallamala Hills of Which of the statements given above is/are
Telangana.
correct?
Which of the statements given above is/are
correct? (a) 2 only
(a) 1 only (b) 1 and 3 only
(b) 2 only
(c) 2 and 3 only
(c) Both 1 and 2
(d) Neither 1 nor 2 (d) 1, 2 and 3

Copyright © by Vision IAS


All rights are reserved. No part of this document may be reproduced, stored in a retrieval system or transmitted
in any form or by any means, electronic, mechanical, photocopying, recording or otherwise, without prior
permission of Vision IAS.
18 www.visionias.in ©Vision IAS
www.UPSCPDF.com
www.UPSCPDF.com www.UPSCPDF.com

VISIONIAS
www.visionias.in
ANSWERS & EXPLANATIONS
GENERAL STUDIES (P) TEST – 3212 (2021)

Q 1.A
• The Madras Native Association (MNA), was founded by activist and merchant Gazulu Lakshimnarasu
Chetty in 1852 to protest the unjust policies of the British regime.
• The MNA represented the landed and business classes of Madras Presidency who had specific
grievances against the Company administration, particularly in the revenue, educational and judicial
spheres. Hence statement 1 is correct.
• The MNA established branches in places like Cuddalore, Tiruchirapalli, Tirunelveli, Salem, and Guntur to
collect information about the grievances of the people in these localities.
• The MNA pointed out how the ryotwari and zamindari systems had reduced the agricultural classes to the
deepest poverty and destitution. It also highlighted the torture tactics resorted to by revenue inspectors to
collect revenue from peasants. This led The Court of Directors to form a commission to investigate the
“alleged cases of torture” by subordinate Indian police officers who collected Government revenue.
• MNA criticized Christian missionary activities and also opposed state support for missionary
activities. It opposed the introduction of the Bible in schools and pleaded for the extension of the
grants-in-aid system to Hindu educational institutions. Hence statement 2 is not correct.

Q 2.B
• DETAILS OF SCHEMES/PROGRAMMES/INITIATIVES UNDERTAKEN BY MINISTRY OF
MINORITY AFFAIRS FOR WELFARE OF MINORITIES:
• (A) Educational Empowerment:
o Scholarship schemes:
§ Pre-Matric Scholarship
§ Post-Matric Scholarship(c) Merit-cum-Means based Scholarship
o Naya Savera – Free Coaching and Allied scheme. The scheme aims to empower the students
belonging to minority communities and prepare them for competitive examinations so that their
participation in government and private jobs improves. The scheme provides financial support for free
coaching to notified minority students in selected coaching institutions.
o Nai Udaan- Support for students clearing Prelims conducted by UPSC, SSC, State Public Service
Commissions, etc., for preparation of Mains Examination (iv) Padho Pardesh- Interest subsidy on
educational loans for overseas studies(v) Maulana Azad National Fellowship (MANF)(vi) Maulana
Azad Education Foundation (MAEF), which implements following two schemes:(a) Begum Hazrat
Mahal National Scholarship for meritorious girls belonging to minorities in class XI and XII(b) Grant-
in-Aid to NGOs
• (B) Area / Infrastructure Development: Multi-sectoral Development Programme (MsDP)
• (C) Economic Empowerment:
o Skill Development:
§ 'Seekho Aur Kamao' (Learn & Earn) – Skill development initiative for minorities.
§ Upgrading Skill and Training in Traditional Arts/Crafts for Development (USTTAD)
§ Nai Manzil- A scheme to provide education and skill training to the youth from minority
communities. Nai Manzil aims to engage constructively with poor Minority youth and help them
obtain sustainable and gainful employment opportunities that can facilitate them to be integrated
with mainstream economic activities
o Concessional loans to minorities through National Minorities Development & Finance Corporation
(NMDFC)
• (D) Women Empowerment:'Nai Roshni'- Scheme for Leadership Development of minority women. The
objective of the scheme is to empower and instill confidence among minority women, including their
1 www.visionias.in ©Vision IAS
www.UPSCPDF.com
www.UPSCPDF.com www.UPSCPDF.com
neighbors from other communities living in the same village/locality, by providing knowledge, tools, and
techniques for interacting with Government systems, banks, and other institutions at all levels.
• (E) Special Needs:
o Hamari Dharohar- To preserve rich heritage and culture of minorities
o Jiyo Parsi- Scheme for containing population decline of the small minority community
• Hence option (b) is the correct answer.

Q 3.A
• Abdul Hamid Lahori (died 1654) was a traveller and historian during the period of Shah Jahan who later
became a court historian of Shah Jahan. He wrote the book Padshahnama also referred to as Badshahnama,
about the reign of Shah Jahan. He has described Shah Jahan's life and activities during the first twenty years
(1627-47) of his reign in this book in great detail.
• Old-age infirmities stopped Lahori from moving through the third decade that was then chronically
compiled and published by Muhammad Waris, a historian.
• Sadullah Khan was the wazir (minister) and scholar in the court of Sahjhan. The third volume of
Badshahnama was later revised by wazir himself.
• Khwaja Abdus Samad is a Persian painter who moved to India in the 16th century. He was one of the
founding masters of the Mughal Miniature tradition. Samad was brought to India by Humayun. His career
under Mughals started from 1550 to 1595.
• Hence option (a) is the correct answer.

Q 4.A
• Hedge fund is a private investment partnership and funds pool that uses varied and complex proprietary
strategies and invests or trades in complex products, including listed and unlisted derivatives.
• Put simply, a hedge fund is a pool of money that takes both short and long positions, buys and sells equities,
initiates arbitrage, and trades bonds, currencies, convertible securities, commodities and derivative products
to generate returns at reduced risk. As the name suggests, the fund tries to hedge risks to investor’s
capital against market volatility by employing alternative investment approaches. Hence option (a) is the
correct answer.
• Hedge fund investors typically include high net worth individuals (HNIs) and families, endowments and
pension funds, insurance companies, and banks.

Q 5.A
• The government of India recently banned more than a hundred Chinese mobile applications including
gaming app PUBG and Chinese search engine app Baidu.
• The government said it has received several complaints from various sources including several reports about
the misuse of some mobile apps available on Android and iOS platforms for "stealing and surreptitiously
transmitting users' data in an unauthorized manner to servers which have locations outside India".
• The government statement read “The compilation of these data, its mining, and profiling by elements hostile
to national security and defense of India, which ultimately impinges upon the sovereignty and integrity of
India, is a matter of very deep and immediate concern which requires emergency measures."
• The government cited that these apps are a threat to the "sovereignty and integrity" of the country. The
government invoked its powers under Section 69A of the Information Technology Act and relevant
provisions under IT Rules 2009 to block these apps. Hence, statement 1 is correct.
• Some argue that those using such apps illegally can face legal consequences but the government has cleared
the air, saying there is no penalty or punishment such individuals.
• Section 69A of the Act provides for a penalty to intermediaries for non-compliance of the blocking order.
However, no penalty is prescribed for individual users of such apps. Hence, statement 2 is not correct.
• However, intermediaries recognised under Section 69A of the IT Act -- social media platforms and others
carrying user-generated content -- will be penalised if they are found flouting the government's order.
Q 6.A
• The Urban Learning Internship Program (TULIP) is a five-year joint initiative of the Ministry of
Housing & Urban Affairs (MoHUA) and the All India Council for Technical Education (AICTE). Hence
statement 2 is not correct.
• It has been conceived pursuant to the Budget 2020-21 announcement by the Finance Minister vision of the
“Government proposes to start a program whereby the urban local bodies across the country would provide
internship opportunities to fresh engineers for a period up to one year.”
• The program aims for providing internship opportunities to fresh graduates in all Urban Local Bodies
(ULBs) and Smart Cities across the country. Hence statement 1 is correct.
2 www.visionias.in ©Vision IAS
www.UPSCPDF.com
www.UPSCPDF.com www.UPSCPDF.com
• It would help enhance the value-to-market of India’s graduates and help create a potential talent pool
in diverse fields like urban planning, transport engineering, environment, municipal finance etc. thus not
only catalysing the creation of prospective city managers but also talented private/ non-government sector
professionals
• The programme does not have any budget of its own, but the 100 smart cities and 4,400 urban local bodies
can use the administrative expenses allocated by the Centre to pay stipends or perks if they wish to. They
are also free to develop their own selection procedures.
• The internships under TULIP are open to Indian Citizens who has to show A Govt. issued identity
proof (Driving License, Election Card, etc.)) and completed a diploma/graduate/postgraduate program.
• Candidates should, however, note that at the time of application they have not crossed 3 years from the date
of results declaration of diploma/graduation.

Q 7.C
• The ‘Infant-Toddler and Caregiver-friendly Neighbourhoods (ITCN) Training and Capacity Building
Programme’ was launched by the National Institute of Urban Affairs (NIUA) under the Ministry of
Housing and Urban Affairs in partnership with Bernard van Leer Foundation (BvLF). The
Programme is designed to help build capacities of city officials and young professionals for developing
young children and family-friendly neighbourhoods within cities in India. Hence statement 2 is not
correct.
• The programme is a continuation of the long-term partnership between NIUA and BvLF to scale up
the efforts and embed the lessons of ITC needs at the neighbourhood level within the city level
programmes. Under the programme, city officials and young professionals are proposed to be skilled
through certified training and capacity building modules. The training is proposed to be delivered
through well-structured training modules, provided online through the National Urban Learning Platform
(NULP), the platform developed for knowledge dissemination by MoHUA and NIUA. Hence statement 3
is not correct.
• The programme is envisioned with the twin objectives: first, embedding the learnings from the inventory
of knowledge developed by NIUA and BvLF within the ongoing and proposed urban development
initiatives at neighbourhood and city level; and second, handholding participants to embed learnings into
the various initiatives of the cities which take into account the everyday needs of young children and
caregivers. In addition, an academic certified course for young professionals has been planned to
sensitize them about the needs of young citizens (0-5 years) in the city and to equip them with adequate
tools for the purpose. Hence statement 1 is correct.
• About NULP:
o The National Urban Learning Platform (NULP) is envisioned as a means of digitally consolidating key
skills and knowledge required by urban stakeholders and making these available to all actors on a
channel of their choice. The NULP will include tools to enable and streamline content creation, content
organisation and management, course building, course management, assessment and certification.

Q 8.B
• When the course of succession is largely driven or determined by internal coactions then such type of
succession is known as Autogenic Succession i.e. the change in both the environment and the community
is brought about by the activities of organisms themselves. Change in the environment for example is
brought about by plant shade or litter input to the soil which in turn will result in a change in structure and
properties of soil leading to change in species composition with time. Hence statement 1 is not correct.
• When the course of succession is controlled by some external driving forces, i.e. the forces beyond the
control of the indigenous organisms or the changes in the environment are caused by some external factor
unaffected by the organism (e.g. storms, fire, etc.), such type of succession is known as allogenic
succession.
• Autogenic succession is largely mediated by direct or indirect competition for niche space, coupled with
changes in the environment resulting from the colonization. Greater is the number of species, greater is
the competition for the niche space. Hence statement 2 is correct.

Q 9.A
• Important facts about Constituent Assembly:
o Elephant was adopted as the symbol (seal) of the Constituent Assembly. Hence statement 1 is
correct.
o Sir B.N. Rau was appointed as the constitutional advisor (Legal advisor) to the Constituent
Assembly. Hence statement 2 is correct.
3 www.visionias.in ©Vision IAS
www.UPSCPDF.com
www.UPSCPDF.com www.UPSCPDF.com
o H.V.R. Iyengar was the Secretary to the Constituent Assembly.
o S.N. Mukerjee was the chief draftsman of the constitution in the Constituent Assembly.
o Prem Behari Narain Raizada was the calligrapher of the Indian Constitution. The original
constitution was handwritten by him in a flowing italic style. Hence statement 3 is correct.
o The original version was beautified and decorated by artists from Shantiniketan including Nand Lal
Bose and Beohar Rammanohar Sinha.
o Beohar Rammanohar Sinha illuminated, beautified and ornamented the original Preamble calligraphed
by Prem Behari Narain Raizada.
o The calligraphy of the Hindi version of the original constitution was done by Vasant Krishan Vaidya
and elegantly decorated and illuminated by Nand Lal Bose.

Q 10.B
• The Ganges canal system irrigates the Doab region between the Ganges and Yamuna rivers. The Ganges
canal is primarily an irrigation system, though it also has a navigation channel and locks. The system
consists of the main canal 437km long and around 6440km of distribution channels. The network has
an upper and a lower canal. It irrigates nearly 9,000km² of fertile farming land in 10 districts of Uttar
Pradesh and Uttarakhand. Hydro-electric plants were later added along with the network. They can
generate around 33MW when running at full capacity.
• The system was devised and built by engineer Proby Thomas Cautley between 1842 and 1854. It
was funded by the British East India Company (BEIC). Hence statement 1 is not correct.
• The BEIC was a company set up to trade with what’s now mainly South East Asia. It ended up mostly
trading with parts of China and seizing control of much of India.
• The canal scheme was built in response to the Agra famine of 1837-38 when over 80,000 people in
northwest India died of starvation after the summer monsoon rains failed. Hence statement 2 is
correct.
• When it opened in 1854, the Ganges canal was the largest and most expensive man-made waterway
in the world.

Q 11.D
• The Wardha Scheme of Education derives its name from the education conference of National Workers
held at Wardha under the presidentship of Mahatma Gandhi organized by congress in 1937.
• The scheme is also known as Nai Talim (Basic Education), Buniyadi Talim (Shiksha).
• The scheme is based on a series of articles Gandhiji wrote in the Harijan newspaper. It was given a concrete
shape by the Committee under the chairmanship of Dr. Zakir Hussain.
• The basic features of the scheme are
o The main principle is learning through activities like handicrafts etc. Hence, statement 1 is
correct.
o Free and compulsory education to be given for 8 years ( from 6 to 14 years). Hence, statement 2
is correct.
o The medium of instructions is to be the mother tongue
o Contact between and community and school through service.
o Providing handicraft training to children. Hence, statement 3 is correct.
o Cleanliness and health, citizenship are to be given sufficient importance.
o It is aimed to provide education that can be self-supporting in later life.

Q 12.C
• The Convention on the Prohibition of the Development, Production, Stockpiling and Use of Chemical
Weapons is a multi lateral international treaty which outlaws the production, stockpiling, and use of
chemical weapons and their precursors. The CWC came into force with effect from 1997.
The Organisation for the Prohibition of Chemical Weapons (OPCW); an intergovernmental
organization based in The Hague, Netherlands is the ‘treaty organisation’ for the CWC. The OPCW bagging
the Nobel Peace Prize of 2013 is in recognition of the Convention as a very effective Disarmament Treaty.
• The National Authority for Chemical Weapons Convention (NACWC) was set up as an office of
the Cabinet Secretariat to fulfill the obligations under the Chemical Weapons Convention (CWC) and to
act as the national focal point for effective liaison with the Organisation for the Prohibition of Chemical
Weapons (OPCW). Hence option (c) is the correct answer.
• In 2000, Chemical Weapons Convention Act was passed by the Parliament which came into force in 2005.

4 www.visionias.in ©Vision IAS


www.UPSCPDF.com
www.UPSCPDF.com www.UPSCPDF.com
Q 13.D
• Aadhaar Authentication means the process by which the Aadhaar number along with the demographic
information or biometric information of a Aadhaar number holder is submitted to the Central Identities Data
Repository (CIDR) for its verification and such repository verifies the correctness, or the lack thereof, on
the basis of the information available with it.
• The Aadhaar number or the authentication thereof shall not, by itself, confer any right of, or be proof of,
citizenship or domicile in respect of an Aadhaar number holder.
• Several requesting entities (or service providers) require individuals to submit their identity proofs that serve
as an enabler for providing consumer services, subsidies or benefits. While collecting such identity proofs,
these service providers face challenges in verifying/validating the correctness of identity information
documents or proofs submitted by individuals.
• The purpose of Aadhaar Authentication is to provide a digital, online identity platform so that the identity
of Aadhaar number holders can be validated instantly anytime, anywhere.
• UIDAI offers Aadhaar-based authentication as a service that can be availed by requesting entities
(government/public and private entities/agencies). Hence, statement 3 is correct.
• This service from UIDAI can be utilized by the requesting entities to authenticate the identity of their
customers/employees / other associates (based on the match of their personal identity information) before
providing them access to their consumer services/subsidies/benefits/business functions/premises.
• The Central / State Government may, for the purpose of establishing the identity of individual as a condition
for receipt of subsidy, benefit or service require that such individual undergo authentication, or furnish proof
of possession of Aadhaar number or in the case of an individual to whom no Aadhaar number has been
assigned, such individual makes an application for enrolment of Aadhaar. Hence, statement 1 is correct.
• In compliance with Aadhaar Act, all requesting entities or service providers shall
o unless otherwise provided in the Act, obtain the consent of an individual before collecting his/her
identity information for the purpose of authentication in such manner as mandated by UIDAI’s policy
and regulations. Hence, statement 2 is correct.
o ensure that the identity information of an individual is only used for submission to the CIDR for
authentication.

Q 14.C
• Osmium, atomic number 76, is a shiny, silver metal that resists corrosion. It is the densest of all the
elements and is twice as dense as lead. Its density of 22.6 g/cm3. Osmium has only a few uses. Hence,
statement 1 is correct.
• Bromine is a fairly abundant element and it is the only non-metal to exist in liquid form at room
temperature. Mercury is the only metal to exist in a liquid state under standard temperature and pressure
conditions. Hence, statement 2 is correct.
• Man-made elements are generally radioactive elements that were not discovered occurring in nature but
as artificially produced isotopes.
o Technetium was discovered by Emilio Segrè in 1937 in Italy. He investigated molybdenum from
California which had been exposed to high energy radiation and he found technetium to be present and
separated it. Today, this element is extracted from spent nuclear fuel rods in tonne quantities.
o In 1937, it became the first element to be discovered by the synthesis in a laboratory. Hence, statement
3 is correct.
• Rare earth elements are a group of seventeen chemical elements that occur together in the periodic table.
o The group consists of yttrium and the 15 lanthanide elements (lanthanum, cerium, praseodymium,
neodymium, promethium, samarium, europium, gadolinium, terbium, dysprosium, holmium, erbium,
thulium, ytterbium, and lutetium). Hence, statement 4 is not correct.
o Scandium is found in most rare earth element deposits and is sometimes classified as a rare earth
element.
o These metals have many similar properties, and that often causes them to be found together in geologic
deposits.
o While Rare Earth elements are used in building consumer electronics, healthcare and transportation,
they are especially important for governments because of their use in manufacturing defence equipment.
At present, China refines approximately 80%-90% of the world’s Rare Earths, thereby having
substantial control over their supply.
Q 15.B
• ‘Bilateral Netting’ refers to net claim or obligations after setting off or adjusting all the claims or
obligations based or arising from mutual dealings between two parties for qualified financial
contracts. Hence statement 1 is correct.
5 www.visionias.in ©Vision IAS
www.UPSCPDF.com
www.UPSCPDF.com www.UPSCPDF.com
• Recently, Parliament enacted the Bilateral Netting of Qualified Financial Contracts Act, 2020 which
seeks to provide a legal framework for bilateral netting of qualified financial contracts (QFC) which
are over the counter derivatives contracts. Multilateral netting has already permitted under Indian
qualified financial contracts. Hence statement 2 is correct
• Bilateral contracts constitute 40 per cent of total financial contracts, while multilateral contracts constitute
60 per cent.
• QFC means any bilateral contract notified as a QFC by the relevant authority which can be Reserve
Bank of India (RBI), Securities and Exchange Board of India (SEBI), Insurance Regulatory and
Development Authority of India (IRDAI), Pension Fund Regulatory and Development Authority (PFRDA)
or International Financial Services Centres Authority (IFSCA).
• As per the Act, qualified financial contracts (QFCs) between two qualified financial market participants
is applicable only where at least one party is an entity regulated by the specified authorities (RBI, SEBI,
IRDAI, PFRDA or the IFSCA). Hence statement 3 is not correct.

Q 16.C
• Position of Women during Sangam Age:
• There is plenty of information in the Sangam literature to trace the position of women during the Sangam
age.
• Women poets like Avvaiyar, Nachchellaiyar, and Kakkaipadiniyar flourished in this period and contributed
to Tamil literature.
• The courage of women was also appreciated in many poems. Karpu or chaste life was considered the highest
virtue of women.
• Love marriage was a common practice.
• Women were allowed to choose their life partners.
• The life of widows was miserable and widow remarriage was not a common practice. Hence option (c) is
the correct answer.
• The practise of Sati was also prevalent in the higher strata of society. The class of dancers was patronized
by the kings and nobles.
• Women in the Sangam period excelled in education and arts. This is testified by many poems
contributed by women poets to the corpus of Sangam literature.
o Women poets like Avvaiyar, Nachchellaiyar and Kakkaipadiniyar flourished in this period and
contributed to Tamil literature.
• Women were engaged in various economic activities such as paddy cultivation, cattle rearing, basket
making, spinning, etc.

Q 17.B
• Silicosis is a long-term lung disease caused by inhaling large amounts of crystalline silica dust, usually over
many years. Silica is a substance naturally found in certain types of stone, rock, sand, and clay. Working
with these materials can create a very fine dust that can be easily inhaled. Once inside the lungs, the dust
particles are attacked by the immune system. This causes swelling (inflammation) and gradually leads to
areas of hardened and scarred lung tissue (fibrosis). Lung tissue that's scarred in this way doesn't function
properly.
• People who work in the following industries are particularly at risk:
o stone masonry and stone cutting – especially with sandstone construction and demolition – as a result
of exposure to concrete and paving materials pottery, ceramics, and glass manufacturing mining and
quarrying and blasting. Hence pair 1 is not correctly matched.
o Chrysotile is serpentine class asbestos. The most common use is corrugated asbestos cement roofing
primarily for outbuildings, warehouses, and garages. It may also be found in sheets or panels used for
ceilings and sometimes for walls and floors. Numerous other items have been made containing
chrysotile including brake linings, fire barriers. Chrysotile asbestos has also produced tumors in animals
and is a recognized cause of asbestosis and malignant mesothelioma in humans. Hence pair 2 is
correctly matched.
o Black lung disease is considered a job-related illness. It affects coal miners, it’s also known as coal
workers’ pneumoconiosis (CWP). As miners breathe in coal dust, particles settle into their airways and
lungs. After they land, healthy lung tissue may try to get rid of them. As their immune system tries to
fight and remove the particles, they can get inflammation. Over time, the inflammation can cause
scarring, which is also known as fibrosis. Hence pair 3 is correctly matched.

6 www.visionias.in ©Vision IAS


www.UPSCPDF.com
www.UPSCPDF.com www.UPSCPDF.com
Q 18.A
• Approximately every 11 years, the sun goes through a natural solar cycle. It is marked by the increase and
decrease of sunspots based upon the magnetic field of the sun.
• According to NASA, the greatest number of sunspots in any given solar cycle is designated as "Solar
Maximum" and opposite to it that is the lowest number is designated as "Solar Minimum". During solar
minimum, the intense activity like sunspots and solar flares subside but that does not mean that the
sun becomes dull. It's just simply changing its form. Hence statement 1 is correct
• During Solar Minimum, there are typically fewer CMEs to cause geomagnetic storms and big Aurora.
However, there are recurrent high-speed solar wind streams that typically cause minor to moderate
geomagnetic storms, so the Northern Lights will still be around. You may just have to travel a bit further
north to see them. Hence statement 2 is not correct.

Q 19.C
• e-SANTA is an electronic marketplace providing a platform to connect aqua farmers and the
buyers. It will enable the farmers to get a better price and the exporters to directly purchase quality products
from the farmers enhancing traceability, a key factor in international trade. Hence option (c) is the correct
answer.
• The term e-SANTA was coined for the web portal, meaning Electronic Solution for Augmenting NaCSA
farmers' Trade in Aquaculture. National Centre for Sustainable Aquaculture (NaCSA) is an extension
arm of Marine Products Export Development Authority (MPEDA), Govt. of India, Ministry of
Commerce & Industry.
• e-SANTA is a completely paperless and end-to-end electronic trade platform between Farmers and
exporters. The farmers have the freedom to list their produce and quote their price while the
exporters have the freedom to list their requirements and also to choose the products based on their
requirements such as desired size, location, harvest dates etc. This enables the farmers and buyers to have
greater control over the trade and enables them to make informed decisions. The platform provides detailed
specification of each product listing and it is backed by an end to end electronic payment system with
NaCSA as an Escrow agent.
Q 20.D
• The Roorkee College was established in 1847 AD as the First Engineering College in the British
Empire. The College was renamed The Thomason College of Civil Engineering in 1854. It was given
the status of University by Act No. IX of 1948 of the United Province (Uttar Pradesh) in recognition of
its performance and its potential and keeping in view the needs of post-independent India.
• Pandit Jawahar Lal Nehru, the first Prime Minister of India, presented the Charter in November
1949 elevating the erstwhile college to the First Engineering University of Independent India. On the
21st of September 2001, the University was declared an institute of national importance, after
passing a bill in the parliament, changing its status from University of Roorkee to Indian Institute of
Technology Roorkee.
• In Bengal, a College called the Calcutta Civil Engineering College was opened at the Writers’
Buildings in November 1856.
• With the establishment of the University of Calcutta on 24 January 1857, the college was affiliated to this
university in May 1857. In 1865, the college merged with Presidency College, Kolkata and from 1865
to 1869 the college functioned as the Civil Engineering Department of Presidency College.
• In 1880, the college was shifted to its present campus at Shibpur, Howrah, and was christened the
Government College, Howrah, in the premises of Bishop’s College. On 12 February 1920, the name was
changed to Bengal Engineering College, Shibpur. The word Shibpur was deleted on 24 March 1921 and
it became Bengal Engineering College. The college was subsequently rechristened Bengal Engineering and
Science University and upgraded to the Indian Institute of Engineering Science and Technology,
Shibpur in 2014.
• In Bombay, the overseers’ school at Pune eventually became the College of Engineering, Pune and
affiliated to the erstwhile University of Bombay (now called University of Mumbai) in 1858. Hence
option (d) is the correct answer.

Q 21.A
• Debt Service Coverage Ratio (DSCR) is a measure of the cash flow available to pay current debt
obligations. The debt-service coverage ratio applies to corporate, government, and personal finance. In the
context of corporate finance, the debt-service coverage ratio (DSCR) is a measurement of a firm's
available cash flow to pay current debt obligations. The DSCR shows investors whether a company has
enough income to pay its debts. Hence statement 1 is correct.
7 www.visionias.in ©Vision IAS
www.UPSCPDF.com
www.UPSCPDF.com www.UPSCPDF.com
• The formula for the debt-service coverage ratio is the ratio of net operating income and the total debt
servicing for the entity.
• Lenders will routinely assess a borrower's DSCR before making a loan. A DSCR of less than 1 means
negative cash flow, which means that the borrower will be unable to cover or pay current debt
obligations without drawing on outside sources - in essence, borrowing more. For example, a DSCR of
.95 means that there is only sufficient net operating income to cover 95% of annual debt payments. Hence
statement 2 is not correct.
Q 22.B
• The International Comparison Program (shortened ICP) is a partnership of various statistical
administrations of up to 199 countries guided by the World Bank. The main partners of this program are
the World Bank, IMF, UN, ADB, OECD etc.
• The Program produces internationally comparable price and volume measures for the gross domestic
product (GDP). Its component expenditures are based on purchasing power parities (PPPs). Hence,
option (b) is the correct answer.
• The program holds surveys collecting price and expenditure data for the entire range of final goods and
services at intervals of some few years (the last two were separated by six years). The surveys cover the
GDPs of countries including their consumer goods, services, government services and capital goods. The
ICP tries to make different countries GDPs comparable by calculating them in PPP both currency converters
and spatial price deflators.
• The responsibility for the ICP within regions is shared between the national and regional agencies which
represent various countries. National statistical offices carry out data collection but regional agencies
provide guidance, collection, coordinating of data and data validation. Regional agencies also put together
and finalize the regional comparisons.
• As per the latest finding of ICP, India has retained its position as the third-largest economy in the world in
terms of purchasing power parity (PPP), behind the US and China. India accounts for 6.7% or $8,051 billion,
out of the world's total of $119,547 billion of global Gross Domestic Product (GDP) in terms of PPP
compared to 16.4 % in case of China and 16.3 % for the US.
• Globally 176 economies participated in the 2017 cycle of ICP. The next ICP comparison will be conducted
for the reference year 2021.

Q 23.B
• The economic conditions during Later Vedic period:
• Iron was used extensively in this period and this enabled the people to clear forests and to bring more land
under cultivation.
• Agriculture became the chief occupation. Improved types of implements were used for cultivation.
Besides barley, rice and wheat were grown. Knowledge of manure was another improvement.
• Industrial activity became more varied and there was greater specialization. Metalwork, leatherwork,
carpentry and pottery made great progress.
• In addition to internal trade, foreign trade became extensive.
• The Later Vedic people were familiar with the sea and they traded with countries like Babylon.
• A class of hereditary merchants (vaniya) came into existence.
• Vaisyas also carried on trade and commerce. They organized themselves into guilds known as ganas.
• Besides nishka of the Rig Vedic period, gold and silver coins like satamana and krishnala were used
as media of exchange. They were not exactly currency coins as a currency system was yet to be established.
• Hence option (b) is the correct answer.

Q 24.A
• The speed of the Earth’s rotation will increase if its mass is brought closer to its axis of rotation. Conversely,
the speed of the Earth’s rotation will decrease if its mass is moved away from the rotation axis. In the case
of Antarctica and Greenland's ice melting and dispersing away from the source creates the mass moving
away from the earth's rotation axis and causes the reduction in the earth's rotation.
• However, the above-mentioned effect couldn't observe when the ice replaces its volume without changing
any mass. Ex: No effect observed when arctic ice capes melted. Hence statement 1 is not correct.
• Melting sea ice, such as the Arctic ice cap, does not change sea level because the ice displaces its volume
only.
• The Gulf stream is a strong ocean current that brings warm air to Northern Europe and relies on dense salty
water from the Arctic in order to function but a flood of freshwater post melting of Arctic ice would dilute
the current and could weaken or even stop it altogether. Without that warm air current, the temperature in
Northern Europe would go much below the freezing point. Hence statement 2 is correct.
8 www.visionias.in ©Vision IAS
www.UPSCPDF.com
www.UPSCPDF.com www.UPSCPDF.com
Q 25.C
• As a federal court, the Supreme Court decides the disputes between different units of the Indian
Federation. More elaborately, any dispute between:
o the Centre and one or more states; or
o the Centre and any state or states on one side and one or more states on the other; or
o between two or more states.
• In the above federal disputes, the Supreme Court has exclusive original jurisdiction. Exclusive means, no
other court can decide such disputes and original means, the power to hear such disputes in the first instance,
not by way of appeal.
• With regard to the exclusive original jurisdiction of the Supreme Court, two points should be noted. One,
the dispute must involve a question (whether of law or fact) on which the existence or extent of a legal right
depends.
• Thus, the questions of political nature are excluded from it. Two, any suit brought before the Supreme Court
by a private citizen against the Centre or a state cannot be entertained under this.
• Further, this jurisdiction of the Supreme Court does not extend to the following:
o A dispute arising out of any pre-Constitution treaty, agreement, covenant, engagement, and or other
similar instruments.
o Inter-state water disputes.
o A dispute arising out of any treaty, agreement, etc., which specifically provides that the said jurisdiction
does not extend to such a dispute.
o Matters referred to the Finance Commission.
o Adjustment of certain expenses and pensions between the Centre and the states.
o Recovery of damages by a state against the Centre.
o Ordinary dispute of Commercial nature between the Centre and the states.
• Hence option (c) is the correct answer.

Q 26.A
• Limbu: The Limbu are indigenous and native people of the Himalayan Limbuwan region of Nepal.
However, a small number of Limbu also live in the western Sikkim, Kalimpong and Western
Bhutan. Hence pair 1 is correctly matched.
• Birhor: Jharkhand is a land of 32 listed Tribes, out of which eight tribes have been categorized as
‘Primitive Tribes’ (Asur, Birjia, Birhor, Kharwar, Parhaiya (Baiga), Saber, Malpahariya & Sauria
Pahariya). The Birhors are one among these primitive tribes and the name is derived from the word ‘Bir’,
meaning jungle and ‘Hor’ meaning people and thus denoting the ‘people of the jungle’. The Birhors of
Jharkhand belongs to the Porto-Australoid stock. They lead a hunter-gatherer life and their mother tongue
Birhor is an Astro-Asiatic language. As per the 2011 census, the total population of Birhors in India is
17,241, out of which about 62% are settled in Jharkhand. Hence pair 2 is correctly matched.
• Bharia: Bharia tribe is one of the especially backward tribe of Madhya Pradesh. This position is given to
Bharia's because still, they are far away from education and modern living. Bharia's consider themselves as
the younger brothers of Gond tribe and hence they consider it to be a duty of them to marry the Gond widow.
Due to the mentioned reasons, we can conclude that both these tribes are interrelated. Location-Bharia Tribe
is found in many districts of Madhya Pradesh and they are as follows: Patal kot area of Chhindwara,
Betul and Hoshangabad. Hence pair 3 is not correctly matched.
• Raji: Uttarakhand is the state of the Central Himalayan region in India, inhabit the five scheduled tribes
–the Tharus, the Jaunsaries, the Buxas, the Bhotias and the Rajis. In Uttarakhand, Raji is one of the
five tribes and one of the two PTGs. Their population is distributed in two districts of Uttarakhand
namely Pithoragarh and Champavat. Their population are scattered in ten villages on hill area of two
districts. Hence pair 4 is not correctly matched.
• Hence option (a) is the correct answer.

Q 27.C
• Most species of dolphins produce two types of sounds, which possibly play the role of communication
signals in their social relationships. These are packs of broadband pulses (Echolcations) and ‘whistles’.
Several species of dolphins of the Kogiidae, Physeteridae and Phocoenidae families and the
Cephalorhynchinae subfamily (Hector's dolphin) do not produce whistles and may communicate by pulsed
sounds. Ganges dolphin produces acoustic signals in the form of long frequency-modulated (FM) acoustic
signals containing a large number of harmonics. Hence pair 1 is not correctly matched.
• Olive Ridley turtle nests along the Indian coastline, which is about 7,500 kilometers long. Their sense of
magnetic fields is clearly very strong. Turtles can sense even the weakest magnetic field. They use Earth's
9 www.visionias.in ©Vision IAS
www.UPSCPDF.com
www.UPSCPDF.com www.UPSCPDF.com
magnetic field and direction of ocean current to navigate between their feeding grounds and nesting
places. Hence pair 2 is not correctly matched.
• Knifefish generate electricity using a specialized organ in its tail containing a small group of cells that can
discharge electricity at frequencies approaching 2000 times per second, the fastest in the animal kingdom.
Unlike their relative the electric eel, these freshwater fish are only weakly electric, not producing enough
charge to stun or kill. Hence pair 3 is correctly matched.
Q 28.D
• An increase in the cash deposit ratio leads to a decrease in the money multiplier. An increase in deposit
rates will induce depositors to deposit more, thereby leading to a decrease in the Cash to Aggregate Deposit
ratio. This will in turn lead to a rise in Money Multiplier. Hence statement 1 is not correct.
• Velocity of Circulation refers to the average number of times a single unit of money changes hands in an
economy during a given period of time. The velocity of circulation is an indicator that transactions between
individuals are occurring more frequently. The velocity of money is typically higher in expanding
economies and lower in contracting economies.
• The velocity of money is equal to GDP divided by money supply. The velocity of money formula shows
the rate at which one unit of money supply currency is being transacted for goods and services in an
economy. Money supply and the velocity of money are inversely proportional. If the money supply in
an economy falls short, then the velocity of money will rise, and vice versa. Hence statement 2 is not
correct.

Q 29.A
• Extirpation (also known as ‘local extinction’) describes the situation in which a species or population
no longer exists within a certain geographical location. Unlike extinction, whereby a species no longer
exists anywhere, extirpation means that at least one other population of the species still persists in other
areas. Hence statement 1 is correct.
• Most species of plants and animals have a number of different breeding populations, which exist either
globally or within a defined region or habitat. This means that when a population ceases to exist in a certain
area, the other populations remain to keep the species extant (still in existence).
• Since the entire species is not extinct, it is possible for populations to recolonize after extirpation.
However, this can lead to a reduction in genetic diversity.
• The habitat ranges of populations may also naturally shift in response to changes in abiotic factors (e.g.,
climatic changes) or changes in biotic factors (e.g., the availability of food, or the presence of
predators and competitive species). Therefore, when a population migrates and no longer exists within a
certain range, it has been extirpated from that area. Hence the statement 2 is not correct.

Q 30.C
• The National Defence Fund was set up to take charge of voluntary donations in cash and kind received for
promotion of the national defence effort, and to decide on their utilisation.
• It is used for the welfare of the members of the Armed Forces (including Para Military Forces) and their
dependents. Hence, statement 1 is correct.
• The Fund is administered by an Executive Committee, with PM as Chairperson, and Defence,
Finance and Home Ministers as Members. Hence, statement 2 is correct.
• Finance Minister is the Treasurer of the Fund. Accounts of the Fund are kept with the Reserve Bank of
India. Hence, statement 3 is not correct.
• The fund is entirely dependent on voluntary contributions from the public and does not get any budgetary
support. Hence, statement 4 is correct.

Q 31.D
• Central Waqf Council is a statutory body under the administrative control of the Ministry of Minority
Affairs. Hence, statements 1 and 3 are correct.
• It was set up in 1964 as per the provision given in the Waqf Act, 1954 as Advisory Body to the Central
Government on matters concerning the working of the Waqf Boards and the due administration of Auqaf.
• The role of the Council was expanded significantly under the provisions of the Waqf (Amendment) Act,
2013. Major functions are:
o To advise Central Government, State Governments, State Waqf Boards on matters concerning the
working of the Boards and the due administration of Auqaf.
o To monitor the implementation of the provisions of Waqf (Amendment) Act, 2013 in States and UTs.
o To render legal advice on protection and retrieval of the Waqf Properties and for removal of
encroachment etc.
10 www.visionias.in ©Vision IAS
www.UPSCPDF.com
www.UPSCPDF.com www.UPSCPDF.com
o To implement the Scheme for Development of Urban Waqf Properties & Identification of potential
Waqf land for development by National Waqf Development Corporation Ltd.
o To implement Educational and Women Welfare Schemes for skill development and to empower the
poor, especially Women. Hence, statement 2 is correct.
o To implement the Scheme of Computerization of the State Waqf Boards records, a Central sector
scheme of Ministry of Minority Affairs.
o To seek the necessary information from the State Government/Boards on the performance of the State
Waqf Boards as per the provision given in the Waqf (Amendment) Act, 2013.
o To take up the Waqf matters with various departments of Central and State Governments such as ASI,
Railways, Revenue and Forest etc.
o To undertake awareness programs to promote the interest of the Council and to sensitize the Waqf
institutions about their new roles and responsibilities.
• The Council consists of Chairperson, who is the Union Minister In-charge of Waqf and such other members,
not exceeding 20 in number, as may be appointed by the Government of India.

Q 32.D
• The Baikal Gigaton Volume Detector (Baikal-GVD) is an underwater neutrino telescope built on
Lake Baikal. The project is aimed at studying the flux of high-energy cosmic neutrinos and searching for
their sources. The telescope is installed 3.5 km offshore in Lake Baikal at the depth of 750-1300 m.
• There aren’t many places on the planet that are as ideal for this type of research as Lake Baikal. The only
two other telescopes that match it in scale are the IceCube Neutrino Observatory in Antarctica and the
ANTARES telescope deep underwater in the Mediterranean Sea.
• Hence option(d) is the correct answer.

Q 33.B
• An e-court or Electronic Court means a location in which matters of law are adjudicated upon, in the
presence of qualified Judge(s) and which has a well-developed technical infrastructure.
• The e-courts project is about providing ICT so as to enable courts to make the justice delivery system
affordable and cost-effective. This would be beneficial for both improving the court processes and rendering
citizen-centric services. E-courts are aimed to make legal processes easier and more user-friendly.
• In an e-court, the entire work is executed digitally, wherein, the information that is shared and generated is
stored as a database and synced to particular software. This software can be accessed by litigants, judges,
and advocates. The primary intention of e-courts is to make the justice delivery system affordable,
transparent, speedy, and accountable by limiting the paper filings.
• The eCourts Mission Mode Project, is a Pan-India Project, monitored and funded by the Department of
Justice, Ministry of Law and Justice, Government of India only for the District Courts across the
country. Hence, statement 1 is not correct.
• The High Court will be the Implementing Agency for implementation of the Project in the Courts under its
jurisdiction. Hence, statement 3 is correct.
• It was conceptualized on the basis of the “National Policy and Action Plan for Implementation of
Information and Communication Technology (ICT) in the Indian Judiciary – 2005” submitted by the
committee, Supreme Court of India.
• It is currently being implemented in two phases.
o Phase I was executed between 2010 and 2015.
o Phase II is being implemented between 2015 and 2020.
§ In phase II, the concept of virtual courts has been introduced. Through this process, it will not be
necessary for lawyers and litigants to attend the court premises thereby reducing footfalls and;
through the Virtual Court the time of the judges also saved and can be utilised for some other
important activity. Hence, statement 2 is correct.

Q 34.C
• Armed Forces Special Powers Act (AFSPA), 1958 grant special powers to the Indian Armed Forces the
power to maintain public order in "disturbed areas". Section 6 of the Act guarantees protection against
prosecution to the armed forces. Before prosecuting army personnel, prior sanction of the central
government must be taken.
• The Act has received criticism from several sections for alleged concerns about human rights violations in
the regions of its enforcement alleged to have happened.
• Supreme Court passed its judgment in 2012 regarding several questions related to prior sanction for
prosecution of army officers under AFSPA.
11 www.visionias.in ©Vision IAS
www.UPSCPDF.com
www.UPSCPDF.com www.UPSCPDF.com
• Is prior sanction required to prosecute army officers for 'any' act committed in the line of duty?
o It held that sanction would not be required in 'all' cases to prosecute an official. The officer only enjoys
immunity from prosecution in cases when he has ‘acted in exercise of powers conferred under the
Act’. Hence, statement 1 is not correct.
o The Court held that the AFSPA empowers the central government to ascertain if action is 'reasonably
connected with the discharge of official duty' and is not a misuse of authority.
o Section 7 of the Act provides protection of persons acting in good faith under this Act.
• At what stage is sanction required?
o The need to seek sanction would only arise at the time of cognizance of the offense. Cognizance is the
stage when the prosecution begins. Sanction is therefore not required during the investigation. Hence,
statement 2 is not correct.
• Is sanction required for a court-martial?
o The Court ruled that there is no requirement of sanction under the Army Act, 1950. Hence, if the Army
chooses, it can prosecute the accused through court-martial instead of going through the criminal
court. Hence, statement 3 is correct.
o If the Army decides on proceedings before the criminal court, the government will have three months
to determine to grant or withhold sanction.
o No prosecution, suit or other legal proceedings shall be instituted, except with the previous sanction of
the Central Government, against any person in respect of anything done or purported to be done in
exercise of the powers conferred by this Act."

Q 35.B
• The Poverty Reduction and Growth Facility (PRGF) is an arm of the International Monetary
Fund which lends to the world's poorest countries. It was created in September 1999, replacing the
Enhanced Structural Adjustment Facility. Hence option (b) is the correct answer.
• It aims at making poverty reduction efforts among low-income members a key and more explicit element
of a renewed growth-oriented economic strategy.
• The PRGF can currently only lend to the poorest countries, which limits the ability of developing
countries with higher income to get low or zero-interest loans from the IMF. The middle-income countries
hard hit by the Covid-19 pandemic are urging IMF to expand the lending to other nations.

Q 36.B
• Third Anglo Mysore War-
o The immediate cause of the war was Tipu's attack on Travancore on December 29, 1789, over a
dispute over Cochin. The Raja of Travancore was entitled to the protection of the English. Thus taking
advantage of the situation, the English, making a triple alliance with the Nizams and the Marathas,
attacked Tipu Sultan.
o On January 29, 1791, Governor-General Cornwallis (1786-1793) himself took over the command
of the British troops. Tipu had to sue for peace and the Treaty of Seringapatnam concluded in March
1792. The treaty resulted in the surrender of nearly half of the Mysorean territory to the victorious allies.
• Second Anglo Maratha War-
o The new Peshwa Baji Rao II was weak and sought the protection of the British through the treaty of
Bassein in 1802. Baji Rao II was restored to Peshwarship under the protection of the East India
Company. However, the treaty was not acceptable to both the Marathas chieftains - the Shindia
and Bhosales. This directly resulted in the Second Anglo-Maratha War in 1803.
o In 1803, Lord Wellesley was the Governor-General (1798-1805).
o Important Treaties during Second Anglo Maratha War-1) Treaty of Rajpurghat between Holkar
and British. 2) Treaty of Surji-Arjanaon between Sindhia and the British. 3) Treaty of Deogaon
between Bhosale and British.
• First Anglo-Sikh War-
o The war began in 1845 after the Sikh Army crossed the river of Sutlej. This was seen as an
aggressive step by the Sikh army which provided justification to the English for the war.
o The Sikhs were forced to sign the Treaty of Lahore on March 8, 1846. During this time, the
Governor-General was Lord Hardinge I (1844-1848).
• Hence option (b) is the correct answer.

12 www.visionias.in ©Vision IAS


www.UPSCPDF.com
www.UPSCPDF.com www.UPSCPDF.com
Q 37.B
• In the southern part of Deccan, in the region of Karnataka are some of the most experimental hybrid style
of Vesara temple architecture are found. It is also known as Karnataka school.
• It combined the features of both Nagara school and Dravidian School of temple architecture.
• Three prominent dynasties that made temples in Vesara styles are Chalukyas, Rashtrakutas and
Hoysala Dynasties. Hence statement 3 is not correct.
• Some of the important features of Vesara style temples are:
o More emphasis is given on vimana and mandapa.
o Open Ambulatory passageway around the main shrine.
o Pillars, doorways and the ceilings were decorated with intricate carvings. Hence statement 2 is
correct.
• Some of the important examples of vesara style temples are Kailashnath temple in Ellora, Doddabasappa
temple at dambal and ladkhan temple at aihole.
• Nayaka school of temple architecture flourished under the nayaka rulers during 16th - 18th century AD.
The art of gopuram reached its climax under the Nayaka rulers. Hence statement 1 is not correct.

Q 38.D
• The Supreme Court introduced the ‘Middle Income Group Scheme’ to provide affordable legal
services where fees would be charged as per the schedule attached to the scheme. Hence, option (d) is the
correct answer.
• The scheme will be administered through a society named ‘Supreme Court Middle Income Group (MIG)
Legal Aid Society’ registered for this purpose.
• The Patron-in-Chief of the society is Chief Justice of India and the Attorney General is its ex-officio Vice
President.
• Its beneficiaries will be litigants in the SC whose gross income is less than Rs. 60,000 per month or Rs. 7.5
lakh per annum.
• Relevant Constitutional Provisions
o The right to free legal aid or free legal service is an essential fundamental right guaranteed by the
Constitution and forms the basis of reasonable, fair and just liberty under Article 21 of the Constitution
of India.
o Article 39-A says that the State shall “ensure that opportunities for securing justice are not denied to
any citizen by reason of economic or other disabilities.

Q 39.B
• Recently, the government set up Rice Export Promotion Forum (REPF) under the aegis of the
Agricultural and Processed Foods Export Promotion Development Authority (APEDA)
o REPF has representations from the rice industry, exporters, officials from APEDA, ministry of
commerce and directors of agriculture from major rice-producing states including West Bengal, Uttar
Pradesh, Punjab, Haryana, Telangana, Andhra Pradesh,
o It will monitor, identify and anticipate developments pertaining to rice production and exports
and recommend necessary policy measures. Hence statement 2 is correct.
• APEDA is a statutory body that was established under the Agricultural and Processed Food Products
Export Development Authority Act, 1985, under the Ministry of Commerce and Industry. Hence
statement 1 is not correct.
o It is mandated with the responsibility of export promotion and development of products such as
Fruits, Vegetables, Meat, Poultry, Dairy Products, Floriculture, Alcoholic and Non-alcoholic
Beverages etc.
• APEDA is mandated; vide APEDA (Amendment) Act, 2009 dated March 6, 2009, for Registration
and Protection of the Intellectual Property Right in respect of special products in India or outside
India.
o The Second Schedule has been added to the APEDA Act containing the names of such products for
which APEDA has been mandated to take these measures. As of now, the Second Schedule has only
one entry i.e. "Basmati Rice". Hence statement 3 is correct.
• APEDA got geographical indication (GI) certification for Basmati. The above GI applied for the region
located in Indo-Gangetic Plains (IGP) below the foothills of the Himalayas, spreads across 7 states. These
are Himachal Pradesh, J&K, Punjab, Haryana, Uttarakhand, Western UP (26 districts) and Delhi.
• APEDA has also been entrusted with the responsibility to monitor the import of sugar as well.

13 www.visionias.in ©Vision IAS


www.UPSCPDF.com
www.UPSCPDF.com www.UPSCPDF.com
Q 40.D
• “National Policy for Rare Diseases 2021” was recently approved by the Ministry of Health and Family
Welfare.
• WHO defines a rare disease as an often debilitating lifelong disease or disorder with a prevalence of
1 or less, per 1000 population. However, different countries have their own definitions to suit their specific
requirements and in the context of their own population, health care system and resources. Hence,
statement 1 is correct.
• Policy objectives:
o improved focus on indigenous research
o screening and detection of rare diseases early at early stages
o lower cost of treatment of rare diseases
o local production of medicines
• The policy has categorised rare diseases into three groups:
o Group 1: Disorders amenable to one-time curative treatment.
o Group 2: Those requiring long-term or lifelong treatment.
o Group 3: Diseases for which definitive treatment is available but challenges are to make optimal patient
selection for benefit, very high cost and lifelong therapy.
• Financial Support:
o Those who are suffering from rare diseases listed under Group 1 will have the financial support of up
to Rs. 20 lakh under the umbrella scheme of Rashtriya Arogya Nidhi.
o Rashtriya Arogya Nidhi: The Scheme provides for financial assistance to patients, living below the
poverty line (BPL) and who are suffering from major life-threatening diseases, to receive medical
treatment at any of the super-specialty Government hospitals/institutes.
o Beneficiaries for such financial assistance would not be limited to BPL families, but extended to about
40% of the population, who are eligible as per norms of Pradhan Mantri Jan Arogya Yojana, for their
treatment in Government tertiary hospitals only.
• It envisages voluntary crowdfunding by setting up a digital platform for voluntary contributions by
individual and corporate donors. Hence, statement 3 is correct.
• It provides for setting up of a national hospital-based registry of rare diseases to ensure that adequate data
and comprehensive definitions of such diseases are available for those interested in research and
development. Hence, statement 2 is correct.
• It aims to strengthen tertiary health care facilities for prevention and treatment of rare diseases through
designating eight health facilities as 'Centres of Excellence' and these will also be provided one-time
financial support of up to Rs. 5 crore for upgradation of diagnostics facilities.

Q 41.D
• In medieval times new devotional forms like the Nath Panthis and Shaktism developed within the Hindu
philosophical traditions.
• Nath Panthis:
o The Nath Panthis claimed a divine origin and according to belief, Adinath or the Shiva was the first
Nath. Hence statement 1 is correct.
o Matsyendranatha and Gorakshanath founded the sect known as the Natha tradition (or
NathSampradaya) or the Goraknathis or the Kanphata Yogis, an order of religious ascetics who stress
the practice of Hatha Yoga.
o Nath Panthis stressed the value of yogic sadhana/meditation in attaining immortality and discarded the
belief that Knowledge of Sanskrit was important to the understanding of spirituality or the absolute
truth. This was the reason they began to preach abstract religious philosophy in the regional
languages. Hence statement 2 is correct.
o The text, Gorakhbodh, suggests that the believer of the Natha order can attain Shivahood not only in
a monastery but even in a market place or under a tree.
o In Bengal, the Nath Panthis were strongly influenced by Shaivism and much later an esoteric offshoot
of their basic beliefs was manifest in the Baul sect.
• Shaktism:
o Worshipping goddesses had always been practised in India since ancient times. This trend continued
taking on newer forms in medieval India with Shaktism evolving as a devotional practice within
Hinduism.
o In its new avatar, the worship of the primordial female power personified as a goddess, in the various
forms of Uma, Kali, Durga and Chandi gained popularity. The worshippers of Shakti or the Universal
female energy were called Shaktas. Hence statement 3 is correct.
14 www.visionias.in ©Vision IAS
www.UPSCPDF.com
www.UPSCPDF.com www.UPSCPDF.com
o In the Shakta scheme of cosmological evolution, the Great Mother had created Brahma, Vishnu and
Shiva.
o In both Shaivism and Vaishnavism, Shiva and Vishnu along with their consorts form an integral part of
the Shakta tradition.
o The movement gained momentum when the Shakta Puranas popularised the legend of the Pithas or
pilgrimages where the goddess was believed to reside. Many Shakti temples, thus, came up all over
India-a tradition that is alive to this day.

Q 42.B
• The 1896 Congress session was held at Calcutta under the presidentship of Rahimtullah Sayani. In this
session, Vande Mataram was sung for the first time by Rabindranath Tagore. Hence statement 1 is correct.
• The 1899 Congress session was held at Lucknow under the presidentship of R.C Dutt. In this session noted
critique of British economic policies, R.C Dutt urged the British government for permanent fixation of land
revenue. Hence statement 2 is correct.
• The last session of Congress before independence was held in 1946 at Meerut. It was held under the
presidentship of J.B Kripalani. Hence statement 3 is not correct.

Q 43.A
• WWF-India has joined hands with the Society for Odonate Studies (SOS) and Thumbipuranam for
the first-ever State Dragonfly Festival in Kerala, christened Thumbimahotsavam 2020. Hence option
(a) is the correct answer.
• This is part of a national dragonfly festival being organized by the WWF India, Bombay Natural History
Society & Indian Dragonfly Society in association with the National Biodiversity Board, United Nations
Environment Programme, United Nations Development Programme, and IUCN – Centre for Environment
Conservation.
• ‘Pantalu’ is the official mascot of the festival which is a genus of a dragonfly. A ‘dragonfly backyard
watch’ has been announced to enhance the participation of people and improve their observation skills, and
this will turn into a citizen science project during COVID-19 restrictions.

Q 44.A
• On the occasion of World Water Day on March 22, a memorandum of agreement was signed between
Union Minister of Jal Shakti and the chief ministers of Madhya Pradesh and Uttar Pradesh to
implement the Ken-Betwa Link Project (KBLP).
• The Ken-Betwa Link Project is the first project under the National Perspective Plan for interlinking of
rivers. Under this project, water from the Ken river will be transferred to the Betwa river. Both these
rivers are tributaries of river Yamuna. Hence statement 1 is correct and statement 2 is not correct.
• The Ken-Betwa Link Project has two phases. Under Phase-I, one of the components — Daudhan dam
complex and its appurtenances like Low Level Tunnel, High Level Tunnel, Ken-Betwa link canal and
Power houses — will be completed. While in the Phase-II, three components — Lower Orr dam, Bina
complex project and Kotha barrage — will be constructed.
• The Ken-Betwa Link Project lies in Bundelkhand, a drought-prone region, which spreads across 13 districts
of Uttar Pradesh and Madhya Pradesh. According to the Jal Shakti Ministry, the project will be of immense
benefit to the water-starved region of Bundelkhand, especially in the districts of Panna, Tikamgarh,
Chhatarpur, Sagar, Damoh, Datia, Vidisha, Shivpuri and Raisen of Madhya Pradesh and Banda, Mahoba,
Jhansi and Lalitpur of Uttar Pradesh.

Q 45.A
• Nicobar Bulbul - Found only on the Nicobar Islands of India, the Nicobar Bulbul is a songbird species in
the bulbul family. Its scientific name is Hypsipetes virescens as long as Ixos is recognized as a distinct
genus. Nicobar Bulbul Hypsipetes nicobariensis in IUCN Red List Criteria has Near Threatened Status.
• The Nilgiri flycatcher (Eumyias albicaudatus) - is an Old World flycatcher with a very restricted range in
the hills of southern India. Nilgiri Flycatcher is a small blue colored bird from the flycatcher family. Its
distribution is confined to the high altitude regions of South India, especially the Nilgiri and Shola
mountains of Western Ghats. The International Union for Conservation of Nature (IUCN) includes it under
the ‘Near Threatened’ category, taking into consideration its population and distribution.
• Western Tragopan - is among the rarest pheasants, endemic to a narrow range in the temperate region of
the Greater Himalaya, between Northwest Frontier Province in Pakistan and western Uttarakhand in India
(Birdlife International 2001). It is also the state bird of Himachal Pradesh. Hence its habitat is not endemic
to India only.
15 www.visionias.in ©Vision IAS
www.UPSCPDF.com
www.UPSCPDF.com www.UPSCPDF.com
• Lesser flamingos inhabit the coastal and inland wetlands of sub-Saharan Africa and India. They breed on
large alkaline and saline lakes, salt pans and coastal lagoon. Lesser flamingos’ numbers are decreasing today
and they are classified as near threatened (NT) on the IUCN Red List.
• Hence option (a) is the correct answer.

Q 46.C
• The State of Food Security and Nutrition in the World is an annual flagship report jointly prepared
by FAO(Food and Agriculture Organisation), IFAD, UNICEF, WFP and WHO to inform on progress
towards ending hunger, achieving food security and improving nutrition and to provide in depth analysis
on key challenges for achieving this goal in the context of the 2030 Agenda for Sustainable Development.
The report targets a wide audience, including policy-makers, international organizations, academic
institutions and the general public. Hence option (c) is the correct answer.
• Current estimates of the report are that nearly 690 million people are hungry, or 8.9 percent of the world
population – up by 10 million people in one year and by nearly 60 million in five years. The number of
people affected by severe food insecurity, which is another measure that approximates hunger, shows a
similar upward trend. In 2019, close to 750 million– or nearly one in ten people in the world – were
exposed to severe levels of food insecurity
• Global Hunger Index is released by Welthungerhilfe. The Global Hunger Index (GHI) is a tool designed
to comprehensively measure and track hunger and undernutrition at global, regional, and national levels. It
is designed to raise awareness and understanding of the fight against hunger, and call attention to those
areas of the world where hunger levels are highest and where the need for additional efforts to eliminate
hunger is greatest.

Q 47.D
• A biosimilar drug, or biosimilar, is a medicine that is very close in structure and function to a biologic drug.
A biologic drug, or biologic, is a drug made from proteins or pieces of proteins (either natural or artificial).
Unlike other drugs, biological drugs must be made in a living system, such as yeast, bacteria, or animal
cells. A biosimilar drug has a structure that is highly similar to, but not exactly the same, as a brand name
biologic drug. Hence statement 1 is not correct.
• Biosimilars and Generic drugs both are as safe and effective as their brand-name drugs. There is no scientific
evidence so far that Biosimilar drugs have a more therapeutic effect and safety features than their Generic
drugs counterpart. Hence statement 2 is not correct.

Q 48.D
• A Non-Banking Financial Company (NBFC) is a company registered under the Companies Act, 1956
engaged in the business of loans and advances, acquisition of shares/stocks/bonds/debentures/
securities issued by Government or local authority or other marketable securities of a like nature, leasing,
hire-purchase, insurance business, chit business but does not include any institution whose principal
business is that of agriculture activity, industrial activity, purchase or sale of any goods (other than
securities) or providing any services and sale/purchase/construction of an immovable property.
• NBFCs that are regulated by RBI are:
o Core Investment Company (CIC): It carries on the business of acquisition of shares and
securities. It is further classified as a systemically Important Core Investment Company (CIC-ND-
SI) whose asset size of Rs 100 crore and holds not less than 90% of its net assets as investments
in equity shares, preference shares, bonds, debentures, debt or loans in group companies.
o it accepts public funds. Hence pair 1 is not correctly matched.
o Micro Finance Institution (MFI): Micro Finance institution is an organization that offers financial
services to low-income populations.
§ Almost all give loans to their members, and many offer insurance, deposit and other services.
§ As per the Economic Survey 2019-20, as of 2016, 97% of the borrowers from Microfinance
Institutions (MFIs) were women, SC/ST and minorities accounted for around 30% and 29%
of the borrowers
§ It provides loan to Household whose annual income is not more than ₹ 1.25 lakh (rural) or ₹ 2 lakhs
(urban) can borrow from MFI.
§ It is jointly regulating by RBI and the Ministry of Corporate Affairs.
o Other NBFCs which are regulated by RBI are Investment and Credit Company, Asset
Reconstruction Companies (ARC), Gold Loan Companies and MUDRA etc.
• Nidhi company: It is a type of company in the Indian non-banking finance sector, recognized under
section 406 of the Companies Act, 2013. Their core business is borrowing and lending money between
16 www.visionias.in ©Vision IAS
www.UPSCPDF.com
www.UPSCPDF.com www.UPSCPDF.com
their members. They are regulated by the Ministry of Corporate Affairs. They have to comply with two
sets of norms, one of Public limited company as per Companies Act, 2013 and another is for Nidhi rules,
2014. Hence pair 2 is not correctly matched.
• Chit fund company: It is a type of saving scheme where a specified number of subscribers contribute
payments in instalment over a defined period. Chit fund is listed in the Concurrent List of the Indian
Constitution. Therefore, both the centre and state can frame legislation regarding chit funds. They are
neither regulated by RBI nor SEBI rather they are registered with the respective state governments.
Hence pair 3 is not correctly matched.
• NBFCs Regulated by SEBI are
o Stock Broker, Mutual Funds (MF) (They pool clients’ money and MF-manager invests it in
shares/bonds using his own discretion & expertise), REITs / InvITs, Investment Banks: (US term)
& Merchant Banking Companies and Venture Capital Fund etc.

Q 49.A
• Octane number”, is a measure of the ignition quality of fuel. The higher this number, the less susceptible is
the fuel to ‘knocking’ when burnt in a standard internal combustion engine. Premium petrol having a
hundred octane number will have the higher anti-knocking property. Hence statement 1 is correct.
• According to a Society of Automotive Engineers’ study increasing the octane number of petrol can result
in reducing fuel consumption of at least 7%. When combined with a small change to the engine, the octane
number increase could reduce CO2 emissions by 20 million tonnes per year from petrol engine vehicles.
• Higher octane also means a reduction of some of the pollutants (volatile organic compounds such as benzene
and particulate matter). Hence statement 2 is not correct.

Q 50.D
• Lushai Hills: The Lushai Hills (or Mizo Hills) are a mountain range in Mizoram and Tripura, India. The
range is part of the Patkai range system and its highest point is 2,157 m high Phawngpui, also known as
'Blue Mountain'. These hills are inhabited by the Lushais and other Mizo tribes, but the population is
extremely scanty.
• Dafla Hills: Daphla (or Dafla) Hills is a tract of hilly country on the border of western Arunachal and
Assam occupied by an independent tribe called Daphla. It lies to the north of the Tezpur and North
Lakhimpur subdivisions, and is bounded on the west by the Aka Hills and on the east by the Abor
Range. The contiguous areas of Pakhui, Sessa orchid and the Eaglenest wildlife sanctuaries are located in
the East and West Kameng districts of western Arunachal Pradesh along the Assam border.
• Barail range: The Barail Range is a tertiary mountain range in Northeast India with an area of
approximately 80,000 ha between Brahmaputra and Barak basins located in Dima Hasao district of
Assam stretch across to Meghalaya on the west, Nagaland and Manipur on the east.
• The Barail Range forms one of the most diverse, but lesser-known, ecosystems of the region. The great
altitudinal variations (from less than 100 m to towering peaks of more than 1,900 m) and the resultant
diversity of vegetation, coupled with a rich faunistic composition, makes the area an ideal choice for wildlife
reserve and an IBA. The Barail is the highest hill range in Assam. It includes the North Cachar Hill
Reserve Forest (RF) of Cachar district, Barail RF of Cachar and North Cachar Hills districts and the
unclassified forests stretching from the Simleng river valley in the west to Laike in the east (in North Cachar
Hills district). The Barail Range is the watershed between the Brahmaputra and Barak rivers. The
terrain ranges from flat and undulating in the river valleys, to mountainous with steep slopes. The highest
peak is 1,959 m near Laike, while Hamplopet (1,867 m) is the second-highest, both of which are outside
reserve forests. The climate is tropical monsoon type.
• Mikir Hills: Mikir Hills are a group of hills located to the south of the Kaziranga National Park, Assam.
The easternmost Meghalaya comprising the detached Mikir Hills is partly isolated being surrounded by
three sides. Karbi plateau or Mikir Hills is known oldest landform in Assam. It is pear-shaped and has an
area of about 7000 sq. km. It is part of the Karbi-Plateau. Its highest peak is Dambuchko.
• Hence option (d) is the correct answer.

17 www.visionias.in ©Vision IAS


www.UPSCPDF.com
www.UPSCPDF.com www.UPSCPDF.com

Q 51.A
• The Directorate General of Foreign Trade (DGFT) is an attached office of the Ministry of Commerce
and Industry and is headed by the Director-General of Foreign Trade. Hence statement 1 is correct.
• This Directorate, with headquarters at New Delhi, is responsible for formulating and implementing the
Foreign Trade Policy with the main objective of promoting India’s exports. The DGFT also issues
scrips/authorization to exporters and monitors their corresponding obligations through a network of 38
regional offices and an extension counter at Indore. Hence statement 2 is correct.
• The Directorate General of Trade Remedies was named in May 2018 as an integrated single-window
agency for providing comprehensive and swift trade defence mechanism in India. Earlier, the Directorate
General of Anti-dumping and Allied Duties (DGAD) dealt with anti-dumping and CVD cases, the
Directorate General of Safeguards (DGS) dealt with safeguard measures and DGFT dealt with quantitative
restriction (QR) safeguards. The DGTR brings DGAD, DGS and Safeguards (QR) functions of DGFT into
its fold by merging them into one single national entity. DGTR now deals with Anti-dumping, CVD and
Safeguard measures. Hence statement 3 is not correct.

Q 52.B
• Guano Islands: South America is very fortunate to possess rich deposits of nitrates in the desert of
Atacama. Nitrate is an important source of Manure and fertilizers. Chile is the largest producer of nitrates.
• Similarly, Guano islands, off the coast of Peru, are extremely fortunate in having the world’s most
concentrated single source of natural manure. These rainless and desert islands are the home of millions
and millions of Guano birds. These birds live entirely on sea fish. It is believed that they consume nearly
five million tonnes of fish in these waters every year. The droppings of the sea birds have been in use for
several hundred years as valuable manure for a variety of crops. Today they are in great demand for
sugarcane and cotton crops. This natural manure contains all the ingredients of plant food that can be readily
assimilated by the plants. Hence option (b) is the correct answer.
Q 53.D
• A biologist has described a Himalayan serow as resembling a cross between a goat, a donkey, a cow, and a
pig. It’s a medium-sized mammal with a large head, thick neck, short limbs, long, mule-like ears, and a coat
of dark hair.
• Previously assessed as ‘near threatened, the Himalayan serow is now been categorized as ‘vulnerable’ in
the IUCN Red List of Threatened Species.
• It is listed under Schedule I of The Wildlife Protection Act, 1972, which provides absolute
protection. Hence statements 1 and 2 are correct.
• Recently Himalayan serow spotted in Spiti valley of Himalayan cold desert National park in December
2020 and in March 2021 it was spotted in Manas National park Assam. Hence statement 3 is correct.

18 www.visionias.in ©Vision IAS


www.UPSCPDF.com
www.UPSCPDF.com www.UPSCPDF.com
Q 54.C
• Charba is the popular folk dance of Himachal Pradesh, performed during the Dussehra festival. Hence,
pair 1 is correctly matched.
• Paika is a martial folk dance performed in the southern parts of Odisha. Paika is a form of a long spear.
the dancers are armed with wooden spears and shields and show off their skills and agility. Hence, pair 2
is also correctly matched.
• Jhoomar (also, jhumar)is performed by tribal Sikhs in Punjab and nearby areas during the harvest season.
It is performed in a circle by professional acrobats. Movement of arms is the most important part, on the
tunes of drums. Hence, pair 3 is not correctly matched.

Q 55.D
• HSRP definition: HSRP is an aluminum-made "high-security registration plate" which is fixed on a
vehicle using two non-reusable locks. A chromium-based hologram of Ashoka Chakra of 20 mm x 20
mm size is applied by hot stamping on the top left-hand corner of the plate in both front and rear
plates to protect against counterfeiting.
• The permanent identification number (PIN) of a minimum of 10 digits is laser branded into the
reflective sheeting on the bottom left-hand side of the registration plate with a letter size of 5 mm. The
HSRP is linked electronically to the vehicle after its affixture on the vehicle on registration. Hence option
(d) is the correct answer.
• Benefits of HSRP:
• The prime reason why HSRP has been made mandatory is that the old number plates are quite easy to
temper with and can be misused by car thieves. However, the HSRP plates come with a non-removable
snap-on lock and are difficult to replace.
• The HSRPs comprises details like engine number and chassis number in a centralised database. The data
helps in identifying a stolen car. The stored data along with a 10 digit PIN helps in identifying a stolen car.
The HSRP plates are only issued once the vehicle owner passes on essential details like engine number and
chassis number.

Q 56.B
• Global Centre for Nuclear Energy Partnership( GCNEP) is an initiative of the Department of Atomic
Energy, Government of India. Hence statement 1 is not correct.
• Government of India has approved the establishment of the Global Centre for Nuclear Energy
Partnership (GCNEP) at village Jasaur Kheri & Kheri Jasaur, near Bahadurgarh, District Jhajjar,
Haryana, in September 2010. GCNEP is the sixth R&D unit under the aegis of the Department of Atomic
Energy (DAE).
• GCNEP will help in capacity building, in association with the interested countries and the IAEA, involving
technology, human resource development, education & training and giving momentum to R&D in enlisted
areas. The main objectives of the center include
o Development of enhanced nuclear safeguards to effectively and efficiently monitor nuclear materials
and facilities.
o Promoting the development of advanced, more proliferation-resistant nuclear power reactors.
o Training manpower in the field of Nuclear Security and Radiological Safety.
o Educating in the field of Advanced Nuclear Energy Systems, Isotopes and Radiation Technologies,
nuclear forensic.
o Establishing accreditation facilities for radiation monitoring.
• Hence statements 2 and 3 are correct.
• The center will house the following 5 schools to carry out its objectives:
o School of Advanced Nuclear Energy System Studies (SANESS)
o School of Nuclear Security Studies (SNSS)
o School on Radiological Safety Studies (SRSS)
o School of Nuclear Material Characterization Studies (SNMCS)
o School for Studies on Applications of Radioisotopes and Radiation Technologies (SARRT)
Q 57.B
• National Pension Scheme (NPS) is based on a contribution basis. it is designed to give retirement income
to senior citizens and to inculcate the habit of saving for retirement amongst the citizens. NPS is regulated
by Pension Fund Regulatory and Development Authority (PFRDA)
• Statement 1 is not correct: Pension returns are market-linked and not provide a fixed return to a
beneficiary. It offers three kinds of funds to subscribers: Equities, Corporate Bonds, Government
Securities.
19 www.visionias.in ©Vision IAS
www.UPSCPDF.com
www.UPSCPDF.com www.UPSCPDF.com
• Statement 2 is correct: All citizens i.e. Unorganized sector workers and private employees, Non-Resident
Indians(NRIs) and OCIs (added recently) till the age of 65 years with bank accounts in India are eligible
under the scheme.
• PFRDA has appointed National Securities Depository Limited (NSDL) as Central Recordkeeping Agency
(CRA). It is the first of its kind venture which will carry the function of recordkeeping, Administration and
Customer Service for all subscribers under the scheme.
Q 58.A
• ‘Tchay-e-gard Shikar’ is a traditional form of fishing in Kashmir. The historical practice is also known as
Shadow fishing. It is specifically associated with Anchar Lake in the Soura area of Srinagar. The fishermen
set out on wooden boats equipped with harpoons. They hide under a cloth or makeshift umbrellas on the
nook of the boat. This causes a shadow on the water that attracts fish. The fishermen strike the approaching
fish with their harpoons.
• This technique of fishing is usually carried out during winter when the fish harvest falls due to lack of water
and freezing of lakes. This is when fish that remain beneath the water surface due to the cold swim towards
the shadows for feeding.
• Hence option (a) is the correct answer.

Q 59.B
• Survey of India (SOI), the national survey and mapping organization under the Ministry of Science and
Technology, was set up in 1767. Hence, statement 1 is not correct.
• In its assigned role as the National Principal Mapping Agency, Survey of India bears a special responsibility
to ensure that the country’s domain is explored and mapped suitably to provide base maps for expeditious
and integrated development and ensure that all resources contribute their full measure to the progress,
prosperity and security of the country.
• SOI has undertaken the creation of a digital cartographic database of topographical maps on various scales.
The digital data is being used by various agencies for planning and GIS applications.
• It also undertakes large scale surveys for various developmental projects including hydroelectric, irrigation,
command area, canal area, cantt. area schemes, etc.
• The Department is responsible to fix the external boundary of India, their depiction on maps published in
the country and also advise on the demarcation of inter-state boundaries. Hence, statement 2 is correct.
• Observation and understanding of special oceanographic characteristics of the Indian Ocean is the prime
focus of National Institute of Oceanography. Hence, statement 3 is not correct.
o The National Institute of Oceanography (NIO) headquartered at Goa, and regional centres at Kochi,
Mumbai and Visakhapatnam, is one of the 37 constituent laboratories of the Council of Scientific &
Industrial Research (CSIR). It was established in 1966 following the International Indian Ocean
Expedition (IIOE) in the 1960s.
o In addition to basic research, the institute carries out applied research sponsored by the industry. These
studies include oceanographic data collection, environmental impact assessment, and modeling to
predict environmental impact.
• Survey of India makes regular observations for Tidal and Sea Surface data, magnetic data that are of
immense value for scientists.

Q 60.B
• "The “miracle of aggregation” is defined as the situation where large numbers of uninformed people balance
each other out when voting. Hence, option (b) is the correct answer.
• Since most people are uninformed, they vote one way or the other, and the few people voting that are
informed tip the tide.
• In other words, the result of uninformed crowds is a wash but a few experts can shift the group to a better
decision. This only works with very large numbers.
• In theory, with the miracle of aggregation, we would get the same outcome if everyone were fully informed
as we would if a very large number of people were uninformed.
• However, there are counterexamples that prove this system does not work.

Q 61.B
• The term “biomimetics” originates from the Greek words “bios” (life) and “mimesis” (to imitate). It is the
study of nature and natural phenomena to understand the principles of underlying mechanisms, to obtain
ideas from nature, and to apply concepts that may benefit science, engineering, and medicines. Nature-
based solutions are emulated to solve the complex problems of the human world. For example shape of
airplanes developed from the look of birds for smooth flying. Hence statement 1 is correct.
20 www.visionias.in ©Vision IAS
www.UPSCPDF.com
www.UPSCPDF.com www.UPSCPDF.com
• Example of Biomimetics: The use of SkinzWraps(a special type of filter) in automobiles reduced car
pollution considerably and increased fuel efficiency by 18%–20%. SkinzWraps, a film inspired by the micro
projections on shark skin to repel germs is an example of Biomimetics. Hence option (b) is the correct
answer.
Q 62.B
• Article 280 of the Constitution of India provides for a Finance Commission as a quasi-judicial body. It is
constituted by the president of India every fifth year or at such an earlier time as he considers necessary.
• The Finance Commission consists of a chairman and four other members to be appointed by the president.
They hold office for such a period as specified by the president in his order. They are eligible for
reappointment.
• The Constitution authorizes the Parliament to determine the qualifications of members of the
commission and the manner in which they should be selected.
• Accordingly, the Parliament has specified the qualifications of the chairman and members of the
commission.
• Finance Commission is required to make recommendations to the president of India on the principles
that should govern the grants-in-aid to the states by the Centre (i.e., out of the consolidated fund of
India).
• The recommendations made by the Finance Commission are only of advisory nature and hence, not
binding on the government. It is up to the Union government to implement its recommendations on
granting money to the states.
• Hence option (b) is the correct answer.

Q 63.A
• Rural Technology Action Group (RuTAG)RuTAG :
• Office of the Principal Scientific Adviser (PSA) to the Government of India conceptualized a mission
called Rural Technology Action Group (RuTAG) with an aim to provide a higher level of science and
technology intervention and support for development and dissemination of appropriate technologies
for rural areas in 2003-04.RuTAGis centered in 7 IITs (Bombay, Delhi, Guwahati, Kanpur, Kharagpur,
Madras, and Roorkee) at present. Hence option (a) is the correct answer.
• Objectives: To identify the technology needs of rural areas, available technology solutions and problems
encountered in adopting the existing technology at grass root levels ; find technology solutions through
government agencies, S&T institutions, S&T NGOs, academic institutes, corporate sectors and
other voluntary agencies; and disseminate refined technologies to rural areas .
• Technology Interventions: The technology interventions of RuTAG are essentially demand driven,
could be for technology upgradation, hi-tech delivery, technology training and demonstration or through
any other innovative method and have focus on problems associated with marginal communities in
rural areas for livelihood creation, drudgery reduction, increase in efficiency/productivity of processes,
provisions of higher income, generation of employment, downsizing of existing technologies, and local
resource management and knowledge generation for further applications in rural context.

Q 64.D
• The Index of Industrial Production (IIP) is a composite indicator that measures the growth rates in different
industry groups of the economy in a fixed period of time. It is compiled and published by National Statistical
Office and is released with a time lag of 6 weeks.
• The scope of IIP as recommended by the United Nations Statistical Office (UNSO) includes manufacturing,
construction, electricity, mining, gas, and water supply. But due to constraints of data availability in India
IIP compilation has excluded construction, gas, and water supply sectors. Hence option (d) is the
correct answer.

Q 65.A
• Saura (also Saora) paintings are made by the Saura Tribe of Odisha. It is a wall mural painting and
is ritualistic in nature.
• A Saora painting is called Idital and the person who creates it is known as the iditalma.
• Saura paintings are done to appease their presiding deity, Edital, who is invoked during all rituals and
celebrations.
• Each painting has a rectangular frame and features icons of deities, or those drawn from nature.
• The paintings are done mostly in white, while the backdrop of the painting is red or yellow. The colours
are extracted from minerals and plants. The human shapes are geometrical and stick-like.
• Hence option (a) is the correct answer.
21 www.visionias.in ©Vision IAS
www.UPSCPDF.com
www.UPSCPDF.com www.UPSCPDF.com
Q 66.A
• The NR6 provides an update of progress in the achievement of 12 National Biodiversity Targets (NBT)
developed under the Convention process in line with the 20 global Aichi biodiversity targets. The report
was submitted to the CBD Secretariat by Environment Minister Harsh Vardhan through online
mode. Hence statement 1 is correct.
• The Report highlights that while India has exceeded/overachieved Two NBTs, it is on track to achieve eight
NBTs and in respect of the remaining two NBTs also, India is striving to meet the targets by the stipulated
time of 2020.
• With well over 20 percent of its total geographical area under biodiversity conservation, India has exceeded
the terrestrial component of 17 percent of Aichi target 11, and 20 percent of corresponding NBT relating to
areas under biodiversity management. Similarly, India has also made noteworthy achievements towards
NBT relating to access and benefit-sharing (ABS) by operationalizing the Nagoya Protocol on ABS. Hence
statement 2 is not correct.

Q 67.D
• The gradual Industrialisation of India created a strong working-class during the early part of the 20th
century.
• The growth of cotton, jute, and iron, and steel industries led to the formation of a strong working class.
There were nearly 2.1 million workers in 1911 in the organized industry.
• This period also witnessed the formation of Trade unions and government and employers felt representation
of labour is necessary for peaceful industrial activity without strikes.
• Government of India Act, 1919 provided for representation of labour in legislative councils and later
extended it to municipalities also.
• To accommodate working-class interests along with the capitalist class Congress in its Amritsar
session in 1919 adopted a resolution urging provincial congress committees to promote labour unions
throughout India.
• Bombay trade disputes Act, 1938 enacted by the Bombay provincial congress government had
provisions like
o Prevention of strikes and lock-outs and to promote the amicable settlement of Trades Disputes in
factories and other industrial establishments.
o if a Union is not recognized it must show 50 percent. membership of the total employed before it
is registered
• Trade Union leaders in India expressed their unanimous condemnation of this Act and Subhas Chandra
Bose, President of the Indian National Congress, also expressed his dissatisfaction with the Act
• In Bombay alone, no less than two hundred thousand workers of all trades came out on the streets in
strike action, in order to demonstrate their emphatic opposition to a measure which is calculated to
severely curtail their liberty of action.
• Hence, all the statements are correct.

Q 68.A
• The Prime Minister, Shri Narendra Modi inaugurated ‘Maitri Setu’ between India and Bangladesh
through a video conference. The Bridge(setu) would be built over Feni river which flows between Indian
boundary of Tripura and Bangladesh. Hence option (a) is the correct answer.
• Due to this bridge over Feni River Agartala will become the nearest city to an international sea port in
India.
• The cost of constructing the bridge, as well as the approach roads to it in both Bangladesh and India, was
borne by India. The Tripura Public Works Department was appointed to execute the project. The bridge
connects Tripura with the Chittagong port in Bangladesh providing landlocked North East India with access
to the sea, and enabling the transport of heavy machines and goods to North East India via Bangladesh.

Q 69.C
• The Brahmi script is the earliest writing system developed in India after the Indus-Valley script. It is one
of the most influential writing systems; all modern Indian scripts and several hundred scripts found in
Southeast and East Asia are derived from Brahmi. Hence statement 1 is not correct.
o Brahmi is usually written from left to right. Hence, statement 2 is correct.
o Brahmi is an abugida, meaning that each letter represents a consonant while vowels are written with
obligatory diacritics called matras.
o Despite the lack of earlier examples, some scholars argued that the Brahmi script had originated earlier
than the 3rd century BCE.
22 www.visionias.in ©Vision IAS
www.UPSCPDF.com
www.UPSCPDF.com www.UPSCPDF.com
o Most examples of Brahmi found in North and Central India represent the Prakrit language.
The Ashokan Inscriptions show some slight regional variations on the Brahmi script.
o The Brahmi script was deciphered in 1837 by James Prinsep.
• The Ashokan inscriptions were composed in the Prakrit language (in Magadhi, the dialect of Prakrit in
Magadha) and written in Brahmi script throughout the greater part of the empire. But in the north-western
part, they appear in Kharosthi script and in Kandahar in Afghanistan, they were written in Aramaic, Greek
script and Greek language. Hence, statement 3 is not correct.

Q 70.B
• The distribution of legislative powers between the Centre and the states is rigid. Consequently, the Centre
cannot delegate its legislative powers to the states and a single state cannot request the Parliament to make
a law on a state subject.
• The distribution of executive power, in general, follows the distribution of legislative powers. But, such a
rigid division in the executive sphere may lead to occasional conflicts between the two. Hence, the
Constitution provides for inter-government delegation of executive functions in order to mitigate rigidity
and avoid deadlock.
• Accordingly, the President may, with the consent of the state government, entrust to that government
any of the executive functions of the Centre.
• Conversely, the governor of a state may, with the consent of the Central government, entrust to that
government any of the executive functions of the state. Hence statement 2 is correct.
• The Constitution also makes a provision for the entrustment of the executive functions of the Centre
to a state without the consent of that state. But, in this case, the delegation is by the Parliament and
not by the president. Hence statement 1 is not correct.
• A law made by the Parliament on a subject of the Union List can confer powers and impose duties on a
state, or authorise the conferring of powers and imposition of duties by the Centre upon a state (irrespective
of the consent of the state concerned). Notably, the same thing cannot be done by the state legislature.
• From the above, it is clear that the mutual delegation of functions between the Centre and the state can take
place either under an agreement or by a legislation. While the Centre can use both the methods, a state can
use only the first method.

Q 71.D
• Recent Context: A petition was filed in Delhi High Court regarding the present recruitment policy for the
President’s Bodyguard. It alleged discrimination in recruitment since the candidates from only three castes
were considered for recruitment to the regiment.
• The President's Bodyguard (PBG) is the oldest regiment of the Indian Army and recently completed 245
years of service.
• It is the senior-most unit of the Army, whose distinction is in its position as ‘Right of the Line’ on all official
and ceremonial occasions. The PBG, thus, takes precedence over all other regiments and corps. Hence,
statement 2 is correct.
• The PBG was raised in 1773 at Benares (now Varanasi) by the then Governor Warren Hastings, with a
strength of 50 handpicked troopers. This nucleus of the Bodyguard was later augmented by another 50
horsemen, provided by Raja Chait Singh of Benares, thus bringing the overall strength of the regiment up
to 100 horses and men by the end of that year. Hence, statement 1 is correct.
• The establishment of the regiment varied through the years, being augmented in times of war and it attained
its maximum strength of 1,929 all ranks, as per the Army List of 1845, just prior to the First Sikh War.
• The PBG continued to be a select Cavalry Unit, primarily for the personal and battlefield security of the
Governor and later Governor General, who often had to personally lead his forces into battle.
• It invites members from only 3 castes — Jats, Rajputs & Jat Sikhs — to apply for the recruitment. Hence,
statement 3 is correct.

Q 72.B
• Vigyan Jyoti programme, a new initiative to encourage girls to take interest in science and build careers,
was launched by the Department of Science & Technology (DST) to create a level-playing field for the
meritorious girls to pursue STEM. Hence statement 1 is not correct and statement 2 is correct.
• Vigyan Jyoti activities include student-parent counseling, visit labs and knowledge centers, partners role
model interactions, science camps, academic support classes, resource material distribution, and tinkering
activities. Online academic support to students includes streaming of video classes, study materials, daily
practice problems, and doubt clearing sessions.

23 www.visionias.in ©Vision IAS


www.UPSCPDF.com
www.UPSCPDF.com www.UPSCPDF.com
• Department of Science and technology has collaborated with IBM to escalate the Vigyan Jyoti
program. Women tech experts working at IBM India will interact inspire girl students to plan for a career
in STEM under the program. This will strengthen DST’s initiative to increase the number of women in
technology fields.
• Vigyan Jyoti initiative by DST focuses on solving the multidimensional problems associated with the
meager representation of women in Engineering and Technology streams in higher education by
building confidence and excitement towards these streams.

Q 73.B
• The national electric grid in India has an installed capacity of 382.15 GW as of 31 March 2021. Renewable
power plants, which also include large hydroelectric plants, constitute 36.8% of India's total installed
capacity. India is the 3rd largest producer and 3rd largest consumer of electricity in the world. Hence
statement 2 is not correct.
• As of July 2020, India has a total Thermal installed capacity of 231.45 GW. Almost 86% of the thermal
power is obtained from coal and the rest from Lignite, Diesel and Gas. The private sector generates 46.9%
of India’s thermal power whereas States and Centre generate 27.9% and 25.3% respectively.
• 100% FDI is allowed in the Power sector for generation from all sources (except atomic energy),
transmission and distribution of electric energy and Power Trading under the automatic route. 49% FDI
allowed in Power Exchanges registered under the Central Electricity Regulatory Commission (Power
Market) Regulations, 2010 under automatic route. Hence statement 1 is correct.
• Hence option (b) is the correct answer.

Q 74.D
• Insects and arachnids are of the same phylum called Arthropoda, which is Greek for “joined foot.” Insects
belong to the class Insecta, and arachnids to the class Arachnida; both classes fall under the Arthropoda
phylum, along with crustaceans (crabs, lobster, and shrimp). Hence statement 1 is not correct.
• Arachnids have only two: the cephalothorax (the head and thorax fused together) and the abdomen. They
have eight legs (four pairs of legs) while insects have six (three pairs of legs). Eight legs are the most
conspicuous features of Arachnids. Besides Arachnids don’t have antennae or wings however almost all the
insects have antennae, and also have wings. Hence statement 2 is not correct.

Q 75.C
• It is a unique breed of camel found only in Kutch (Gujarat). Hence Statement 1 is correct.
• They have the special ability to survive on both, dry land and in the sea. They can swim in seawater, up to
three kilometers, and feed on saline plants and mangroves. Hence statement 2 is correct.
• It is one of the most preferred choices of graziers in the arid coastal region of Kachchh. Kharai was
recognized as a separate breed in 2015 by the Indian Council of Agricultural Research. Hence statement 3
is correct

Q 76.B
• Rapoport's Rule states that the species at higher latitudes tend to have a larger latitudinal range. Suggested
explanations for Rapoport's rule include decreased land area closer to the equator, increased interspecific
competition from greater species diversity at lower latitudes, differential rates of glacial driven extinction
at higher latitudes, and increased climatic variability and thermal tolerance at higher latitudes. Hence pair
1 is not correctly matched.
• Bergmann’s Rule states that the body size is large in cold climates and small in warm climates. Large
bodies have smaller surface area to volume ratios. Hence pair 2 is correctly matched.
• Gloger's Rule posits that darker birds are found more often in humid environments than in arid ones,
especially in the tropics. Accordingly, desert-inhabiting animals tend to be light-colored. This rule is also
true for certain mammalian groups, including humans. Hence pair 3 is correctly matched.

Q 77.D
• Recently India announced that the Gandhi Peace Prize for the past two years will be conferred on late
Omani Sultan Qaboos bin Said Al Said and Bangladeshi leader Sheikh Mujibur Rahman for their
contributions to social and political transformation through non-violent methods.
• A special exception was made for the award to be conferred on the two leaders, as the selection
procedure for the prize does not allow it to be awarded posthumously. Hence statement 2 is not
correct.

24 www.visionias.in ©Vision IAS


www.UPSCPDF.com
www.UPSCPDF.com www.UPSCPDF.com
• The prize is given for social, economic and political transformation through non-violence and other
Gandhian methods. It can be conferred upon an individual, an organisation and can also be shared between
two individuals. Hence statement 3 is correct.
• The award includes an amount of Rupees 1 crore, a citation, a plaque and a traditional handicraft or
handloom item. The award is open to all persons irrespective of caste, creed , nationality and sex.
Hence statement 1 is correct.

Q 78.B
• NITI Aayog, in partnership with Bill and Melinda Gates Foundation and Centre for Social and
Behaviour Change, Ashoka University, launched Poshan Gyan, a national digital repository on
health and nutrition. Hence statement 1 is not correct and statement 2 is correct.
• The Poshan Gyan repository was conceptualized as a resource, enabling search of communication materials
on 14 thematic areas of health and nutrition across diverse languages, media types, target audiences and
sources. Content for the repository was sourced from the Ministries of Health and Family Welfare and
Women and Child Development and developmental organizations. The website provides an intuitive
interface (multi-parametric search, multiple downloads at a point in time, easy sharing of materials via
social media and easy viewing on any type of smartphone).
• The crowdsourcing feature of the site allows anyone to submit information for inclusion into health
practices, which can be used to disseminate nationwide solutions that have proved successful in local
contexts, such as the use of a traditional grape-based concoction that was recently found to be efficacious
in combating anaemia in Tamil Nadu.

Q 79.D
• Atal Innovation Mission (AIM), NITI Aayog launched AIM-PRIME (Program for Researchers on
Innovations, Market-Readiness & Entrepreneurship), an initiative to promote and support science-
based deep-tech startups & ventures across India. Hence statement 1 is correct.
• AIM has joined hands with Bill & Melinda Gates Foundation (BMGF) to launch this nationwide
program which will be implemented by Venture Center - a non-profit technology business
incubator. Hence statement 3 is correct.
• The first cohort of the program is open to technology developers (early-stage deep tech start-ups, and
scientists/ engineers/ clinicians) with strong science-based deep tech business ideas. The program is also
open to CEOs and Senior incubation managers of AIM Funded Atal Incubation Centers that are
supporting deep tech entrepreneurs. Hence statement 2 is not correct.
• The benefits of this program are aimed at addressing specific issues through training and guidance over a
period of 12 months. Candidates selected for the program will get access to in-depth learning via a
comprehensive lecture series, live team projects, exercises, and project-specific mentoring.
• The AIM-PRIME program is specifically tailored for the rapid scaling up of deep-tech science ventures in
India, providing not just the necessary intellect and support but also the exposure to the participants they
rightly deserve.

Q 80.C
• Context: The scientists at CSIR-Institute of Himalayan Bioresource, Palampur (IHBT), are on a mission to
grow Asafoetida (heeng) in the Indian Himalayas. The first sapling has been recently planted in Himachal
Pradesh’s Kwaring village in Lahaul valley. Ferula asafoetida is a herbaceous plant of the Umbelliferae
family.
• It is a perennial plant whose oleo gum resin is extracted from its thick roots and rhizome. The plant stores
most of its nutrients inside its deep fleshy roots. Asafoetida is endemic to Iran and Afghanistan, the main
global suppliers. It thrives in dry and cold desert conditions. It is very popular in India as a spice
crop. Hence statement 1 is correct.
• Heeng is not cultivated in India. Government data states that India imports about 1,200 tonnes of raw
heeng worth Rs 600 crore from Iran, Afghanistan, and Uzbekistan. Hence statement 2 is correct

Q 81.A
• Invest India is the National Investment Promotion and Facilitation Agency of India and act as the first point
of reference for investors in India.
• Invest India was formed in 2009 under Section 25 of the Companies Act 1956 for the promotion of
foreign investment. It was established as a non-profit venture. It functions under the Department of
Industrial Policy and Promotion, Ministry of Commerce and Industry. Hence statements 1 and 2 are
correct.
25 www.visionias.in ©Vision IAS
www.UPSCPDF.com
www.UPSCPDF.com www.UPSCPDF.com
• The current shareholding pattern of Invest India is 51% of Industry Associations (i.e. 17% each of FICCI,
CII & NASSCOM) and the remaining 49% of Central and 19 State Governments. Hence statement 3 is
not correct.
• As the national investment promotion and facilitation agency, Invest India focuses on sector-specific
investor targeting and the development of new partnerships to enable sustainable investments in India. In
addition to a core team that focuses on sustainable investments, Invest India also partners with substantial
investment promotion agencies and multilateral organizations.
• In November, 202, UNDP and Invest India have launched the SDG Investor Map for India, laying out
18 Investment Opportunities Areas (IOAs) in six critical SDG enabling sectors, that can help India push the
needle forward on Sustainable Development.

Q 82.D
• Bhaskaracharya was one of the prominent Indian mathematicians of the 12th century and astronomer,
who wrote the book ‘Sidhantshiromani’. The book is divided into four sections:
o Lilavati - dealing with arithmetics
o Beejganita - dealing with algebra
o Goladhyaya - about spheres
o Grahaganita - mathematics of planets
• About Bhaskaracharya: Bhaskara was one of the many outstanding mathematics hailing from South India.
Born in 1114 CE in Karnataka, he composed a four-part text entitled the Siddhanta
Siromani. Included in this compilation is the Bijaganita (or Beejaganita), which became the standard
algebra textbook in Sanskrit. It contains descriptions of advanced mathematical techniques involving both
positive and negative integers as well as zero, irrational numbers. It treats at length the “pulverizer”
(kuttakara) method of solving indeterminate equations with continued fractions, as well as the so-called
Pell's equation (vargaprakrti) dealing with indeterminate equations of the second degree.
• He also wrote on the solution to numerous kinds of linear and quadratic equations, including those involving
multiple unknowns, and equations involving the product of different unknowns. In short, he wrote a highly
sophisticated mathematical text that preceded by several centuries the development of such techniques in
Europe, although it would be better to term this a rediscovery since much of the Renaissance advances of
mathematics in Europe was based upon the discovery of Arab mathematical texts, which were in turn highly
influenced by these Indian traditions. Hence option (d) is the correct answer.

Q 83.B
• Next Generation Incubation Scheme (NGIS) is a comprehensive incubation scheme with a vision to drive
the rise of India as a Software Product Nation.
• NextGen Startup Challenge Contest (CHUNAUTI - Challenge Hunt Under NGIS for Advanced
Uninhibited Technology Intervention) is an online Challenge under NGIS further boost startups and
software products with a special focus on Tier-II towns of India. Hence statement 2 is correct.
• The programme was launched by the Ministry of Electronics and Information Technology. The
government has earmarked a budget of Rs. 95.03 Crore over a period of three years for this programme. It
aims to identify around 300 startups working in identified areas and provide them seed fund of upto Rs.
25 Lakh and other facilities. Hence statement 1 is not correct.
• The startups selected through Chunauti will be provided various support from the Government
through Software Technology Parks of India centers across India. They will get incubation facilities,
mentorship, security testing facilities, access to venture capitalist funding, industry connect as well as
advisories in legal, Human Resource (HR), IPR and Patent matters.

Q 84.A
• Vijnana Bharati (Vibha) and Global Indian Scientists’ and Technocrats’ Forum (GIST) have come
together to launch a mission named "AAHAR KRANTI" with the motto of`Good Diet-Good Cognition’.
The `Aahaar Kranti’ movement is designed to address the peculiar problem being faced by India and the
world called `hunger and diseases in abundance’. Hence statement 2 is not correct
• The movement proposes to address the current situation of hunger in abundance by working to rouse the
people to the values of India’s traditional diet, to the healing powers of local fruits and vegetables, and to
the miracles of a balanced diet. It will renew the focus on nutritionally balanced diets. The programme will
focus on training teachers, who, in turn, will pass on the message to the multitudes of students, and through
them to their families and finally the society at large. Such a strategy was adopted for the eradication of
Polio and it turned out to be a grand success. Hence statement 1 is not correct, since the movement is
about balanced diet and traditional food habits.
26 www.visionias.in ©Vision IAS
www.UPSCPDF.com
www.UPSCPDF.com www.UPSCPDF.com
• In terms of objectives, it will seek to promote better awareness, better nutrition and better agriculture; the
messages will be imparted through the curriculum in the form of `what’s and `why’s of nutrition, or through
the forms of games or as instructions such as `how to’; and the content will be provided both online and
offline and in all vernacular languages besides English and Hindi to reach out to as many as possible.
• India has the unique advantage of having the knowledge of Ayurveda. It is time to also apply this rich
knowledge of Ayurveda-based nutrition to practice. The movement will work on this too. Hence
statement 3 is correct
• GIST is an autonomous platform created by Vijnana Bharati to link global Indians with the under
privileged (40 crore) masses in the country, mostly residing in more than three lakh villages. GIST was
officially launched by Dr. APJ Abdul Kalam on 26 December 2010.

Q 85.C
• India plans to launch a new orbiter to Venus in 2024. Shukrayaan orbiter will be the first mission to Venus
by the India Space Research Organization (ISRO) and will study the planet for four years. It is set to launch
on India's GSLV Mk II rocket.
• French space agency (CNES) announced it would also fly an instrument on Shukrayaan. The Venus Infrared
Atmospheric Gases Linker (VIRAL) is a collaboration with Russian federal space agency Roscosmos for
this purpose. Hence statement 1 is correct.
• Phosphine, a toxic gas made up of one phosphorus and three hydrogen atoms (PH3), commonly produced
by organic life forms but otherwise difficult to make on rocky planets, was discovered in the middle layer
of the Venus atmosphere. Therefore it is an important biomarker. Hence statement 2 is correct.

Q 86.A
• Mesabi range: Mesabi Range is the largest iron ranges in northern Minnesota, U.S. (the others are
Vermilion and Cuyuna). It is the largest of four major iron ranges in the region collectively known as
the Iron Range of Minnesota.
• Highveld: About half of the gold ever produced in the world has been mined on the Highveld in South
Africa since 1880. The largest deposits are located in the Witwatersrand, which centres on Johannesburg,
with smaller deposits in the northern Free State near Welkom and Virginia. The gold in this basin occurs in
the coarse-grained conglomerates forming the upper portions of the “Witwatersrand Supergroup” that hosts
roughly 7,000m of sedimentation.
• Kinta Valley: Kinta Valley in Perak, Malaysia, has historically been the most productive tin-
producing district in the world. The valley is formed by the Kinta River, a tributary of Sungai Perak, which
flows between the Main Range and the Kledang Range. It forms the largest tin field along the Siamese-
Malayan peninsula tin belt.
• Chuquicamata: It is one of the largest open-pit copper mines and the second deepest open-pit mine in the
world, is located 1,650 km north of Santiago, Chile. The mine, popularly known as Chuqui, has been
operating since 1910. Chuquicamata is an opencast copper mine, where relatively poor copper ore is mined
in impressive huge amounts. Hence option (a) is the correct answer.

Q 87.D
• Acids are a substance with particular chemical properties including turning litmus red, neutralizing alkalis,
and dissolving some metals. An acid can donate an H+ ion and can remain energetically favourable after
a loss of H+. They have a pH value of less than 7.
• Bases is a substance able to accept a hydrogen ion from an acid. They can donate hydroxide ions (OH-) in
solution. They turn red litmus blue and have a pH value greater than 7.
• A special type of substances is used to test whether a substance is acidic or basic. These substances are
known as indicators.
o Litmus is a natural indicator, turmeric is another such indicator.
o There are many other natural materials like red cabbage leaves, coloured petals of some flowers such
as Hydrangea, Petunia and Geranium, which indicate the presence of acid or base in a solution. These
are called acid-base indicators or sometimes simply indicators.
o Olfactory indicators are substances that change their smell when mixed with an acid or base, for
example, onion, vanilla and clove.
• Hence (d) is the correct answer.

27 www.visionias.in ©Vision IAS


www.UPSCPDF.com
www.UPSCPDF.com www.UPSCPDF.com
Q 88.C
• Delimitation literally means the act or process of fixing limits or boundaries of territorial constituencies in
a country or a province having a legislative body. The job of delimitation is assigned to a high power body.
Such a body is known as Delimitation Commission or a Boundary Commission. In India, such Delimitation
Commissions have been constituted 4 times – 1952, 1962, 1972, 2002.
• The 42nd Amendment Act of 1976 froze the allocation of seats in the Lok Sabha to the states and the
division of each state into territorial constituencies till the year 2000 at the 1971 level.
• This ban on readjustment was extended for another 25 years (ie, up to the year 2026) by the 84th
Amendment Act of 2001, with the same objective of encouraging population limiting measures.
• The 84th Amendment Act of 2001 also empowered the government to undertake readjustment and
rationalization of territorial constituencies in the states on the basis of the population figures of the 1991
census.
• Later, the 87th Amendment Act of 2003 provided for the delimitation of constituencies on the basis of the
2001 census and not the 1991 census. Hence statement 1 is correct.
• After every census, a readjustment is to be made in (a) allocation of seats in the Lok Sabha to the states, and
(b) division of each state into territorial constituencies.
• Parliament is empowered to determine the authority and the manner in which it is to be made.
• Accordingly, the Parliament has enacted the Delimitation Commission Acts in 1952, 1962, 1972, and 2002
for this purpose. Hence statement 2 is correct.

Q 89.D
• Recently Parliament enacted the National Bank for Financing Infrastructure and Development
(NaBFID) Act, 2021 which laid down the foundation for the establishment of a statutory institution,
National Bank for Financing Infrastructure and Development (NBFID) which was announced in the
Budget speech by Finance Minister as the principal development financial institution (DFIs) for
infrastructure financing.
• It will have both developmental as well as financial objectives.
o Developmental objectives include facilitating the development of the market for bonds, loans, and
derivatives for long-term infrastructure financing.
o Functions of NBFID include:
§ extending loans and advances for infrastructure projects,
§ taking over or refinancing such existing loans,
§ attracting investment from private sector investors and institutional investors for infrastructure
projects,
§ organising and facilitating foreign participation in infrastructure projects as the principal
development financial institution (DFIs) for long-term infrastructure financing. Hence statement
1 is correct.
o Financial objectives will be to directly or indirectly lend, invest, or attract investments for
infrastructure projects located entirely or partly in India. The central government will prescribe
the sectors to be covered under the infrastructure domain. Hence statement 3 is correct.
o NBFID may raise money in the form of loans or otherwise both in Indian rupees and foreign
currencies, or secure money by the issue and sale of various financial instruments including bonds
and debentures. NBFID may borrow money from: (i) central government, (ii) Reserve Bank of
India (RBI), (iii) scheduled commercial banks, (iii) mutual funds, and (iv) multilateral institutions
such as World Bank and Asian Development Bank. Hence statement 2 is correct.
o It have authorised share capital of one lakh crore rupees.
§ Shares of NBFID may be held by: (i) central government, (ii) multilateral institutions, (iii)
sovereign wealth funds, (iv) pension funds, (v) insurers, (vi) financial institutions, (vii) banks, and
(viii) any other institution prescribed by the central government.
§ Initially, the central government will own 100% shares of the institution which may
subsequently be reduced up to 26%.

Q 90.A
• Monetary policy refers to the use of monetary instruments under the control of the central bank to
regulate magnitudes such as interest rates, money supply and availability of credit with a view to
achieving the ultimate objective of economic policy.
• The Reserve Bank of India (RBI) is vested with the responsibility of conducting monetary policy. This
responsibility is explicitly mandated under the Reserve Bank of India Act, 1934.
• There are several direct and indirect instruments that are used for implementing monetary policy.
28 www.visionias.in ©Vision IAS
www.UPSCPDF.com
www.UPSCPDF.com www.UPSCPDF.com
• Corridor is one of the monetary instruments that refers to the area between the lower reverse repo rate
and the upper ceiling rate of the Marginal Standing Facility (MSF) rate. Therefore, it determines the
corridor for the daily movement in the weighted average call money rate.
• Other instruments of monetary policy are Cash Reserve Ratio (CRR), Statutory Liquidity Ratio (SLR, Repo
Rate, Reverse Repo Rate. Open Market Operations (OMOs): and Market Stabilisation Scheme
(MSS). Hence option (a) the correct answer.

Q 91.D
• The definition of services trade under the General Agreement on Trade in Services (GATS) is four-
pronged, depending on the territorial presence of the supplier and the consumer at the time of the
transaction.
• (Mode 1 — Cross border trade):
o It refers to cover services flows from the territory of one member into the territory of another. For eg.
Business Process Outsourcing, KPO or LPO services. Here, it’s in India’s interest to push for
liberalization given its large human resource pool and competitive IT industry.
• (Mode 2 — Consumption abroad).
o It refers to situations where a service consumer (e.g. tourist or patient) moves into another
member's territory to obtain a service. Tourism and travel-related services include services provided
by hotels and restaurants (including catering), travel agencies and tour operator services, tourist
guide services and other related services.
• (Mode 3 — Commercial presence):
o It refers to the service supplier of one member establishes a territorial presence, including through
ownership or lease of premises, in another member's territory to provide a service (e.g. domestic
subsidiaries of foreign insurance companies or hotel chains)
• (Mode 4 — Presence of natural persons): It refers to the presence of persons of one WTO member in
the territory of another for the purpose of providing a service. It does not concern persons seeking
access to the employment market in the host member, nor does it affect measures regarding citizenship,
residence or employment on a permanent basis. Hence option (d) is the correct answer.

Q 92.C
• Peaty and Marshy Soils: Peaty soils originate in humid regions as a result of the accumulation of large
amounts of organic matter in the soils. These soils contain a considerable amount of soluble salts and 10-
40 per cent of organic matter. Hence statement 1 is correct.
• Soils belonging to this group are found in the Kottayam and Alappuzha districts of Kerala where it is
called Kari. Marshy soils with a large proportion of vegetable matter also occur in the coastal areas of
Odisha and Tamil Nadu, Sunderbans of West Bengal, in Bihar and Almora district of Uttarakhand. Hence
statement 3 is correct.
• The peaty soils are black, heavy and highly acidic. Hence statement 2 is correct. They are deficient in
potash and phosphate. Most of the peaty soils are underwater during the rainy season but as soon as the
rain ceases, they are put under paddy cultivation.

Q 93.C
• Mega food park scheme under Ministry of Food Processing and Industries aims to provide modern
infrastructure facilities for the food processing. It is based on a cluster-based approach along the value chain
from the farm to the market. The Mega Food Park Scheme (MFPS) aims at providing modern infrastructure
facilities for food processing along the value chain from farm to market.
• Statement 1 is correct: It aims to link agricultural production to the market by bringing together farmers,
processors and retailers. It establishes a mechanism to ensure maximization of value addition and
minimisation of wastages, thus improving farmers income.
• Statement 2 is not correct: It is implemented by a Special Purpose Vehicle (SPV) which is a Body
Corporate registered under the Companies Act. State Governments or their entities can avoid setting
separate SPV for the implementation of the scheme.
• Statement 3 is correct: This scheme is now a component of the new Central Sector Umbrella Scheme–
Pradhan Mantri Kisan Sampada Yojana (PMKSY). The scheme provides for a capital grant at the rate of
50 percent of the project cost (excluding land cost) in general areas and at the rate of 75 percent of the
project cost (excluding land cost) in difficult and hilly areas i.e. North East Region including Sikkim, J&K,
Himachal Pradesh, Uttarakhand and ITDP notified areas of the States subject to a maximum of Rs.50 crores
per project. The requirement of land for establishing the CPC is minimum 50 acres either by purchase or on
lease of at least 75 years for the Central Processing Centre of the Mega Food Park.
29 www.visionias.in ©Vision IAS
www.UPSCPDF.com
www.UPSCPDF.com www.UPSCPDF.com
• Key Project Components include:
o Enabling Basic Infrastructure –Internal Roads, drainage, water supply, electricity supply including
captive power plant, ETP & STP, weighbridges etc.
o Core Processing –Sorting & grading, packaging, warehouses & Specialized storage, Pre-cooling and
Ripening chambers, IQF, Cold chain infrastructure, Steam generation, QC Lab etc.
o Standard Design Facility Sheds for Micro and Small Enterprises (MSEs) – plug and play facility
for small units.
o Non – Core Infrastructure - Admin buildings, training centres, canteen, workers’ hostel, trade/display
centre etc.

Q 94.D
• Ratoon cropping: It is growing a fresh crop from the stubbles or suckers of the plant crop without
replanting. It is also referred to as stubble cropping, re-harvesting, second crop, etc. Ratooning is the
agricultural practise of harvesting a monocot crop by cutting most of the above-ground portion but leaving
the roots and the growing shoot apices intact so as to allow the plants to recover and produce a fresh crop
in the next season. Ratoon cropping is an old system that has been practised for many years, especially in
the Tropics.
• Ratoon cropping is used extensively in sugarcane, bananas and plantains, pineapple, forage crops and
minor fibre crops such as abaca and ramie; and to a limited extent in rice, sorghum, pigeon pea,
pineapple, lucerne fodder, and some vegetables. Hence option (d) is the correct answer.
• Advantage: Ratoon cropping is an age-old practice with several advantages. Most importantly it is a cost-
effective practice as it saves seed material and to some extent, labour and inputs. It makes efficient use of
the growing seasons and facilitates crop intensification, and thus helps in improving agricultural
productivity.
• Disadvantage: Ratoon crops cannot be perennially renewed and may be harvested only for a few seasons,
as a decline in yield tends to occur due to increased crowding, damage by pests and diseases, and decreasing
soil fertility.

Q 95.B
• To address the issue of ensuring environmentally sound management of hazardous waste for the safety of
health and environment during handling of such waste Hazardous and Other Wastes (Management and
Transboundary Movement) Rules, 2016 were notified under Environment (Protection) Act, 1986. The Rules
lay down the procedure towards this process by providing provisions for authorization of hazardous waste
generating and units using hazardous waste.
• The major salient features of Hazardous and Other Wastes (Management and Transboundary Movement)
Rules, 2016 include the following:
o the ambit of the Rules has been expanded by including ‘Other-Waste’.;
o waste management hierarchy in the sequence of priority as prevention, minimization, reuse, recycling,
recovery, co-processing; and safe disposal being incorporated;
o the process of import/export of waste under the rules has been streamlined by simplifying the procedure
and by revising the list of waste regulated for import/export;
o separate schedule has been introduced which comprises waste such as metal scrap, paper waste and
various categories of electrical and electronic equipment for re-use purpose exempted from the
need of obtaining Ministry’s permission;
o the list of waste prohibited for import has been revised by the inclusion of the following items: waste
edible fats and oils of animals or vegetable origin, household waste, critical care medical
equipment, solid plastic wastes, other chemical wastes especially in solvent form.
• Hence, option (b) is the correct answer.

Q 96.A
• During the Quit India movement on February 10, 1943, Gandhiji commenced a fast of 21 days in Jail. It
was his answer to the slogan 'Do or Die' slogan in the face of government pressure on Mahatma Gandhi to
condemn the violence of the people.
• Mahatma Gandhi refused to condemn the violence of the people and held the colonial government
responsible for it. Hence the fast was directed against the colonial government's repressive acts and
arrest of thousands of congressmen.
• By early May of 1944 Gandhiji's health deteriorated quickly and the colonial government considered
it a serious risk if Gandhi would die as a prisoner in British hands.

30 www.visionias.in ©Vision IAS


www.UPSCPDF.com
www.UPSCPDF.com www.UPSCPDF.com
• Hence, Viceroy ordered for the release of Gandhi on 6th May 1944 unconditionally on medical
grounds.
• Hence only statement 1 is correct.

Q 97.A
• The Principal Scientific Adviser to the Government of India launched the “MANAS” App to
promote wellbeing across age groups. MANAS which stands for Mental Health and Normalcy
Augmentation System was endorsed as a national program by the Prime Minister’s Science, Technology,
and Innovation Advisory Council (PM-STIAC).
• MANAS is a comprehensive, scalable, and national digital wellbeing platform and an app developed to
augment the mental well-being of Indian citizens. MANAS App integrates the health and wellness
efforts of various government ministries, scientifically validated indigenous tools with gamified interfaces
developed/researched by various national bodies and research institutions.
• It was jointly executed by NIMHANS Bengaluru, AFMC Pune, and C-DAC Bengaluru.
• MANAS is based on life skills and core psychological process, with universal accessibility, delivering age-
appropriate methods and promoting a positive attitude focusing on wellness. Catering to the overall
wellbeing of people of all age groups, the initial version of MANAS focuses on promoting positive mental
health in the age group of 15-35 years.
• Hence option (a) is the correct answer.

Q 98.C
• Amrabad Tiger Reserve lies in the Nallamala hills, Telangana. Earlier, it was part of the Nagarjunasagar-
Srisailam Tiger Reserve. Amrabad Tiger Reserve is India’s second-largest tiger reserve, next only to
Nagarjunasagar Srisailam Tiger Reserve (Andhra Pradesh and Telangana).
• It harbors great biodiversity, consisting of around 70 species of mammals, more than 300 hundred avian
varieties, 60 species of reptiles, and thousands of insects, all supported and nourished by more than 600
different plant species. It has a large presence of the Chenchu tribe.
• According to the All India Tiger Estimation Report 2018, Amrabad Tiger Reserve along with Kawal Tiger
Reserve both in Telangana have 26 Tigers. Recently it was in news for loss of habitat and biodiversity due
to uranium mining and forest fires. Hence both statements 1 and 2 are correct.

Q 99.D
• GAGAN is an acronym for GPS Aided GEO Augmented Navigation. It is a Space Based Augmentation
System (SBAS) jointly developed by ISRO and AAI. It is a system to improve the accuracy of a GNSS
receiver by providing reference signals.
• The GAGAN is implemented to provide required accuracy, continuity, availability, and integrity as per
ICAO SARPS to enable users/aircraft to rely on GPS for all phases of flight.
• GAGAN Signal-in-Space is available to users from two geostationary satellites (GSAT-8 and GSAT-10)
and third geo-satellite GSAT-15 is in the process of integration with GAGAN system.
• Who benefits from GAGAN?
o Aviation: GAGAN is designed to assist navigation both en-route as well as during landing. The
potential benefits will assist air traffic control to cope with increased traffic as well as improving
performance while maintaining safety and reducing the infrastructure needed on the ground.
o Maritime: GAGAN can be used for navigation operations, traffic management, seaport operations,
casualty analysis and offshore exploration and the exploitation of fisheries.
o Land transport: GAGAN can be utilized to manage land transport in India, whether it is by road or
railways. It will increase both the capacity and the safety of land transport. Companies operating
transport services need to know where their vehicles are at all times, as do other public services such as
police, ambulance and taxi.
o Diverse potential uses: GAGAN can help farmers in aerial crop spraying or precision farming. GAGAN
is being used for forest and wildlife resource management system using a receiver designed for
specifically this purpose called SXTreo T51.
o Hence, option (d) is the correct answer.

Q 100.A
• The three Jewels (Triratna) embraced under Buddhism are :
o Buddha - the Enlightened One
o Dhamma - Teachings of Buddha
o Sangha - The Monastic Order
31 www.visionias.in ©Vision IAS
www.UPSCPDF.com
www.UPSCPDF.com www.UPSCPDF.com
• Out of the three jewels or triratnas, the concept of Sangha (the Order) was initiated by Buddha at the time
of giving the first sermon at Sarnath. Hence statement 1 is not correct.
• The early Buddhist texts were compiled in Pali. The Pali canons are called the Tripitaka (Three
Baskets). They are Vinaya Pitaka, Sutta Pitaka and Abhidhamma Pitaka. Vinaya Pitaka deals with
monastic rules and moral disciplines. Sutta Pitaka dwells upon discourses and teachings of Buddha.
Abhidhamma Pitaka expounds on Buddhist philosophy. Hence statement 2 is correct.
• The fourth Buddhist council was convened in Kashmir under the patronage of Kanishka (first century
AD) and under the chairmanship of Vasumitra who was helped by Asvaghosha (author of
Buddhacharita). It resulted in the division of all the Buddhists into two major sects, with Sarvastivadins
(popular in Kashmir and Mathura regions) and Mahasanghikas together forming the Mahayanists
(followers of the Greater Vehicle), and the rest, including Sthaviravadins, forming the Hinayanists
(followers of the Lesser Vehicle). Hence statement 3 is not correct.

Copyright © by Vision IAS


All rights are reserved. No part of this document may be reproduced, stored in a retrieval system or transmitted
in any form or by any means, electronic, mechanical, photocopying, recording or otherwise, without prior
permission of Vision IAS.

32 www.visionias.in ©Vision IAS


www.UPSCPDF.com

You might also like